Archer Review Day 1

Ace your homework & exams now with Quizwiz!

The nurse is caring for a teenager who is recovering from a tonsillectomy. The nurse walks into the room and sees the client eating chips and salsa from a Mexican restaurant. Which response by the nurse is most appropriate?

"Those chips are really hard on the back of your throat where you had your surgery. I'm worried they could cause you to bleed if they damage your incision site. Let's get something softer for you to eat right now."

The primary healthcare provider (PHCP) prescribes azithromycin 500 mg, daily for a client with sepsis. The medication label reads 500 mg of azithromycin mixed in 250 mL of 0.9% saline. The drop factor is 15 gtts/mL and it is to be infused over 60 minutes. The nurse sets the flow rate at how many drops per minute? Round your answer to the nearest whole number. Fill in the blank.

63 gtts

If the nurse does not intervene, the client runs the highest risk for hypovolemic shock as evidenced by the client's vital signs

Although this client did ingest alcohol, he is alert and completely oriented, and delirium tremens are not a concern. Perforation is a concern associated with appendicitis - not a complication common with pancreatitis. The client's vital signs are concerning because he is exhibiting signs of hypovolemic shock. This life-threatening complication of acute pancreatitis calls for aggressive fluid repletion. Establishing peripheral vascular access and initiating the prescribed saline bolus is essential. The client is tachycardic and his heart rate is increasing. Thus, calls for the nurse to act. It is imperative for the nurse to medicate the client for their pain, but pain medication will not resolve the tachycardia that the client is experiencing. A CT scan will help diagnose pancreatitis, and hyperglycemia is likely with acute pancreatitis, but these do not prioritize the client's need for rapid fluid repletion.

A client scheduled for abdominal surgery has just signed the informed consent form. Five minutes later, he tells the nurse "I can't wait to go to the Bahamas. Tell the captain to wait for me!" What is the most appropriate action of the nurse?

Assess the client's level of consciousness

The ICU nurse is caring for a sedated patient on a pressure-cycled ventilator. The ventilator alarm is beeping persistently despite the patient's civil status and stable vitals. What is the most appropriate action for the RN to take first?

Check tubing for holes or kinks

The emergency department (ED) nurse is caring for a client who has overdosed on lorazepam. The nurse anticipates the primary healthcare provider (PHCP) will prescribe which medication?

Choice A is correct. Flumazenil is the antidote for benzodiazepine (BZD) overdose. Lorazepam is a benzodiazepine, so the nurse expects to administer Flumazenil to this patient with BZD overdose. Choice B is incorrect. Phenylephrine is a decongestant used to treat stuffy nose and sinus congestion caused by the common cold, hay fever, or other allergies. There is no indication to give this medication in the case of a benzodiazepine overdose. Epinephrine is a catecholamine that increases the heart rate and blood pressure. This medication is commonly given intramuscular (IM) for anaphylaxis. Naloxone is the antidote for opioid overdose. Lorazepam is a benzodiazepine, not an opioid. Naloxone would reverse an overdose caused by morphine, fentanyl, oxycodone, or other opioid medications.

The nurse is caring for a 30-year-old client who has developed iron-deficiency anemia during pregnancy. Which complication would this client be at an increased risk for due to iron deficiency anemia?

Choices A, B, and D are correct. During pregnancy, there is an increased demand for oxygen to supply both the mother and the developing fetus. Iron deficiency anemia occurs as a result of insufficient amounts of iron (needed to make hemoglobin) to meet oxygen demand. Iron deficiency anemia is associated with an increased risk for low birth weight, preterm delivery, and perinatal mortality. Choice C is incorrect. Iron deficiency anemia in pregnancy is not associated with an increased risk of developing gestational diabetes. Choice E is incorrect. Placenta previa is when the placenta attaches low in the uterus and cover part or all of the cervix. This has no association with iron deficiency anemia.

When orienting an older patient to the safety measures in his hospital room. What is the priority component of this admission routine?

D. Explain how to operate the call light

The nurse enters a client's room and finds the client lying on the floor, appearing unresponsive. The nurse should initially

D. Shout the client's name Choice D is correct. Although the client appears unresponsive, the "true unresponsiveness" can only be determined upon adequate assessment. Therefore, the first action the nurse should take is to check if the client is alert/arousable. The nurse can assess this by shouting the client's name (assessing for a response to verbal stimulation) and tapping the patient/performing a sternal rub (assessing for a response to physical stimulation). If the client is unresponsive despite these measures, the nurse should proceed to carry out other interventions. Among the focused assessment options provided, assessing responsiveness is the fastest and the priority action. Choice A, B, and C are incorrect. The nurse should first assess the client before initiating a code blue. Following an assessment of the client's responsiveness, the nurse should assess respiration/breathing. If the client is found to be unresponsive but breathing, a rapid response should be called. If the client is not breathing, a code blue should be initiated. Pulse must be assessed; however, the nurse's correct sequence of actions is first to assess the client's responsiveness, check for breathing, and assess for the presence of a pulse.

The nurse is preparing a presentation on Cushing's disease. It would be correct if the nurse states that Cushing's disease is caused by

D. increase in the secretion of adrenocorticotropin hormone (ACTH). Choice D is correct. Primary Cushing's disease is characterized by hypersecretion of ACTH from the pituitary gland, which is usually due to a pituitary adenoma. This causes the client to experience multisystem manifestations such as sodium and water retention leading to weight gain, elevated blood glucose, delayed wound healing, and increased gastric acid secretion. Choices A, B, and C are incorrect. Destruction of pancreatic beta-cells is a statement describing the pathophysiology of diabetes mellitus. An excessive discharge of Serum T3 and T4 describes the pathophysiology of hyperthyroidism. A decrease in the secretion of androgens and glucocorticoids is cardinal to adrenal insufficiency (Addison's disease).

The nurse is caring for a hospitalized 18-month-old toddler client. Which is the most appropriate play activity for the nurse to offer the toddler client?

Finger paints

A nurse is in charge of a client scheduled for a liver biopsy at 0800. In preparation for the procedure, the nurse should do which of the following?

Inform the client that their vital signs will be closely monitored after the procedure. Choice D is correct. The client's vital signs will be monitored before, during, and after the procedure. One of the primary reasons for monitoring the vital signs post-procedure is to monitor the client for signs and symptoms of bleeding and/or shock following the procedure.

Which of the following are appropriate nursing interventions to prevent aspiration after a child has vomited?

Position the child on their side. Suction the mouth to remove vomitus. Choices A and B are correct. Positioning the child on their side will prevent aspiration and maintain a patent airway (Choice A). Suctioning the mouth will remove any further vomitus keeping the mouth clean and preventing aspiration (Choice B). Choice C is incorrect. It is not safe to offer the child a sip of water at this time as they may aspirate on thin liquids. The child first needs to be assessed and the problem identified before it is decided that they are safe for oral intake. Chocie D is incorrect. Although it is an appropriate nursing intervention to assess the character and amount of emesis, this does not do anything to prevent aspiration after the child has vomited. Choice E is incorrect. A child that is vomiting may benefit from an antiemetic, but this does not prevent aspiration in the child who has already vomited. Prevention of aspiration in infants and young children include: ✓ feeding in upright or prone position ✓ avoid feedings directly before laying flat for sleep Antiemetic medication therapy for pediatrics is ondansetron, which is a serotonin receptor antagonist.

Complete the diagram by dragging from the choices below to specify what condition the client is most likely experiencing, two (2) actions the nurse should take to address that condition, and two parameters the nurse should monitor to assess the client's progress

Potential Conditions The client is most likely experiencing a cerebral vascular accident (CVA). The client's history of atrial fibrillation substantially increases the risk for an ischemic stroke. Considering the sudden evolution of symptoms, this stroke is most likely embolic (which is a form of an ischemic stroke). The signs and symptoms of expressive aphasia, no motor function on the left side, and left-sided facial drooping are cardinal symptoms of a CVA. Bell's palsy produces facial paralysis but not speech aphasia or unilateral paralysis. A complex partial seizure is unlikely as it has been 45 minutes since the symptom onset, and a complex partial seizure does not sustain for that amount of time. Action to Take An immediate priority for a client experiencing a CVA is transporting the client to a CT scan to determine if the CVA is hemorrhagic or ischemic. The nurse should also obtain laboratory work, including capillary blood glucose, as hypoglycemic symptoms mimic stroke symptoms. However, in this case, considering the severity of symptoms, hypoglycemia is highly unlikely. An LP is not indicated because the suspicion is not infectious. An MRI will be completed during the hospital stay, but the national standard is to have the client from the door to CT scan less than twenty-five minutes. Manual blood pressure is unnecessary, considering hypertension is likely related to an ischemic stroke. Parameters to Monitor The parameters to monitor include a neurological assessment which includes the Glasgow Coma Scale. Additionally, hypertension is expected during a stroke, and permissively, it is preferred to keep the blood pressure elevated to allow for perfusion to the affected area of the brain. The target blood pressure for an ischemic stroke is 185/110 mmHg or less. Hypertension is not usually treated until the SBP is 220 mmHg. Visual acuity is not a standard monitoring parameter in a stroke and is completed for any ophthalmic trauma using a Snellen chart. Monitoring the client's gait and orthostatic blood pressure could be harmful as the client is likely to fall considering her left-sided paralysis. These are two assessments not completed during an acute CVA. Additional Information - Strokes associated with atrial fibrillation have an abrupt onset. These strokes are ischemic and are caused by an embolus from the left atrial appendage. The FAST assessment (facial drooping, arm weakness or drift, slurred speech, time of last known well) is a rapid screening tool for a stroke. Other assessments include the Glasgow Coma Scale and the NIH Stroke Scale. The immediate priority for caring for a client with a suspected stroke is to assess their airway, breathing, circulation and conduct a Glasgow Coma Scale assessment coupled with an NIH Stroke Scale. Pertinent laboratory work should be obtained, including capillary blood glucose to rule out hypoglycemia. The client should be NPO until a bedside swallow evaluation can be completed.

Prior to administering the fentanyl, the nurse should obtain vital signs. After administering the medication and prior to leaving the room, the nurse should raise the client's upper side rails.

Prior to administering the fentanyl, the nurse should obtain vital signs. After administering the medication and prior to leaving the room, the nurse should raise the client's upper side rails.

The nurse is caring for a client with a hyphema. The nurse should plan to take which action?

Shield the affected eye. Choice A is correct. The initial nursing priorities for a hyphema are shielding the affected eye and raising the head-of-the bed to 30 degrees. Choices B, C, and D are incorrect. Placing a client supine would aggravate the injury. The purpose of raising the head-of-the-bed to 30 degrees is because it promotes the settling of blood in the anterior chamber away from the visual axis. Cold compression of the eye would not be helpful. This compression may raise intraocular pressure which would be contraindicated. Aspirin and NSAIDs should be avoided because of their platelet inhibition which will promote more bleeding. A hyphema is an ocular emergency that has been caused by blood in the anterior chamber. This injury results from trauma and should be addressed promptly. Initial nursing actions include: Elevation of the head of the bed to 30 degrees. This will keep the blood below the visual axis. Application of an eye shield to the affected eye. This will prevent further injury. Prescribed pain medication that should not include aspirin or NSAIDs. Educating the client to avoid any activity to raise the intraocular pressure such as bending at the waist, vomiting, or coughing.

The nurse is taking care of a 9-year-old boy undergoing testing for acute myeloid leukemia (AML). She is assisting with the client's positioning for a lumbar puncture. Which of the following positions is appropriate?

Side lying Side-lying (lateral recumbent) is the most appropriate position for a lumbar puncture (LP). The client's legs are flexed at the knee and pulled towards the chest, while the upper thorax is curved forward in an almost fetal position. A pillow may be placed under the client's head and/or between the legs. This position will allow the health care provider to identify the lumbar vertebrae and insert the needle into the subarachnoid space at the L3-4 or L4-5 interspace. The lateral recumbent position is preferred over the upright position because it allows for accurate measurement of the cerebrospinal fluid (CSF) opening pressure. An upright or sitting position may be used for the LP when the client's lateral position is not feasible.

Which assessment finding requires immediate follow-up? The client's

Signs of appendix rupture include pain increasing with coughing that is relieved by bending the right hip (or knee) or sudden pain relief. A fever is expected with appendicitis, along with nausea and vomiting. While the client's sexual activity requires follow-up concerning a potential pregnancy, this is not life-threatening, whereas ruptured appendicitis leads to septic shock.

The nurse is caring for a client who is recovering from surgery. Which assessment data would suggest that the client's pain is not well controlled? Select all that apply.

Tachypnea Nausea Mydriasis Increased blood glucose Choices A, C, D, and E are correct. A client experiencing acute pain will have activation of the sympathetic nervous system, therefore, causing signs and symptoms such as: Nausea, vomiting Diaphoresis Increased pulse Tachypnea Increased blood glucose Increased blood pressure Dilated pupils (mydriasis) Choice B is incorrect. The activation of the sympathetic nervous system is associated with acute pain. Bradycardia is not a finding consistent with this system's activation, as tachycardia would be the likely finding.

The nurse is planning a staff developmental conference about confidentiality. Which of the following scenarios should the nurse include as a violation of client confidentiality?

Telling a police officer who brought a client into the emergency department (ED) the urine drug screen results

The nurse is evaluating the progress of a completely paraplegic female client with a C6-C7 spinal cord injury. Which indicator signifies that the client is improving in physical therapy?

The client can control the motorized wheelchair. Choice A is correct. A C6-C7 spinal cord injury (SCI) can still retain some ability to extend shoulder, arms, and fingers with compromised dexterity in the hands and fingers. The client showing that she can maneuver a wheelchair indicates that she has progressed in therapy. Rehabilitation often will focus on learning to use the non-paralyzed portions of the body to regain varying levels of autonomy. Upon successful treatment, survivors of injuries at the C6/C7 level may be able to drive a modified car with hand controls. The C6 and C7 cervical vertebrae (and the C8 spinal nerve) form the lowest levels of the cervical spine and directly impact the arm and hand muscles. The C6/C7 injury has the potential to change everything below the top of the ribcage, resulting in quadriplegia or paraplegia. Physical therapy is an essential part of recovery. The patient will need to maintain any function not lost by the cord damage, as well as try to regain function. In acute rehabilitation of C6/C7 SCI patients, the focus is on strengthening the upper extremities to the maximal level in patients with complete paraplegia. Empowering exercises for shoulder rotation are proposed for using crutches, swimming, electric bicycles, and walking. At the end of the acute phase, strong upper extremities are needed for the independent transfer from the bed. For this purpose, active and resistance exercises to strengthen the muscles of the upper extremity should be initiated at the earliest possible period. The wheelchair is an essential tool for SCI patients to be mobile and participate in social life. Choices B and C incorrect. A client with C6-C7 Spinal cord injury with complete paraplegia loses control over leg and foot movement completely. These statements by the client indicate that she may need counseling regarding coping with her injuries. Choice D is incorrect. The client who loses control of their bladder may regain function again sometime after injury. However, this is not dependent on physical therapy and does not indicate a positive response to physical therapy.

The nurse plans care for this client

The nurse anticipates that the physician will order a complete blood count (CBC) and a cobalamin (Vitamin 12) level to confirm a pernicious anemia diagnosis. If the client does indeed have pernicious anemia, the CBC will show megaloblastic anemia. Specifically, the macrocytic red blood cells (increased mean corpuscular volume) and low reticulocyte count (immature red blood count). The cobalamin (Vitamin 12) level would be low. A head and neck CT scan is unnecessary or used in diagnosing pernicious anemia. This is also true for an unnecessary urine analysis.

The nurse is most concerned that the client has ruptured appendix because pain that is worsened with coughing.

The nurse is most concerned about a ruptured appendix because it is the most life-threatening and the most likely diagnosis based on the client's increased pain with coughing, guarding of the abdomen, and fever. Paralytic ileus is unlikely because the client's localized abdominal pain and no abdominal distention were noted in the assessment. Her sexual activity would not be an immediate concern even if pregnancy was possible because it would not lead to immediate complications.

The primary healthcare provider (PHCP) prescribes 500 mL of 0.9 saline to infuse over four hours via intravenous micro drip tubing. The nurse sets the flow rate at how many drops per minute? Fill in the Blank. Round your answer to the nearest whole number.

The primary healthcare provider (PHCP) prescribes 500 mL of 0.9 saline to infuse over four hours via intravenous micro drip tubing. The nurse sets the flow rate at how many drops per minute? Microdrip tubing is utilized for precise fluid administration and is likely to be used for infants or children. The drop factor with this tubing is always 60 gtts/mL

The pediatric nurse is reading the chart for a newly admitted child suffering from intussusception. The nurse knows that this disorder is characterized by:

The telescoping of one area of the intestines into another Choice A is correct. Intussusception generally occurs as a result of a blockage in the intestines, which results in the telescoping of one portion of the bowel into another part of the colon. This disorder occurs more frequently in children, often males. Choice B is incorrect. Intussusception is the telescoping of the intestine and does not have to do with alveoli. Choice C is incorrect. Twisted bowel is simply a twist or loop in the gut and is not known as intussusception. Choice D is incorrect. Rectal prolapse occurs from chronic constipation and weakened anal sphincter.

The nurse is caring for a client who sustained a full-thickness burn to his anterior torso, back, and bilateral anterior arms. Using the rule of nine's, calculate the total body surface area (TBSA) burned. Fill in the blank.

This client sustained a 45% TBSA burn ➢ Anterior torso, 18% ➢ Entire back, 18% ➢ Bilateral anterior arms, 4.5% each x 2 = 9%

A client presents to the emergency department (ED) with a suspected ectopic pregnancy. The nurse anticipates which diagnostic test will confirm this finding?

Transvaginal ultrasound Choice C is correct. An ectopic pregnancy (EP) is a medical emergency. The imaging of choice is a transvaginal ultrasound, as this type of ultrasound may visualize an extrauterine gestational sac with a yolk sac or embryo (with or without a heartbeat). Choices A, B, and D are incorrect. Non-stress testing is utilized in the third trimester, where an EP is typically recognized around six or eight gestational weeks. Radiography does not provide a sufficient ability to visualize an embryo. Doppler transducer for fetal heart rate would simply discern if there is a fetal heart rate and not where the embryo is located. ✓ An ectopic pregnancy (EP) is an extrauterine pregnancy. ✓ Almost all ectopic pregnancies occur in the fallopian tube, but other possible sites include cervical, interstitial, hysterotomy (cesarean) scar, ovarian, or abdominal. ✓ Manifestations of an EP include unilateral abdominal (pelvic) pain, vaginal bleeding, and a positive pregnancy test. ✓ Rapid management is needed because life-threatening intraabdominal bleeding may occur. ✓ For EP's that have not ruptured and the woman is stable, methotrexate may be used. If that is not the case, surgical management will be necessary.

Which nursing assessment finding are consistent with hypocalcemia? Select all that apply.

Trousseau's sign Numbness and tingling of the fingers and toes Chvostek's sign Choices A, D and E are correct. Chvostek's sign is an indication of hypocalcemia. This sign is positive if the patient's upper lip twitches upon tapping over a branch of the facial nerve on the same side (Choice A). Trousseau's sign is also an indication of hypocalcemia. Trousseau's is positive if a carpopedal spasm is observed upon inflating a blood pressure cuff past the systolic blood pressure ( Choice D). Other signs and symptoms of hypocalcemia may include muscle cramps, numbness and tingling in the fingers and toes, tetany, seizures, and cardiac arrhythmias (Choice E). Choice B is incorrect. Grey-Turner's sign indicates abdominal pathology, not hypocalcemia. Grey-Turner's sign refers to ecchymosis/ bruising around the flanks and takes about 24-48 hours to develop after the inciting event. The causes of Grey-Turner's sign include retroperitoneal hemorrhage and severe acute pancreatitis. Choice C is incorrect. Homan's sign indicates a deep vein thrombosis ( DVT), not hypocalcemia. Homan's sign is positive when there is deep calf pain and tenderness while extending the leg straight and dorsiflexing the foot. The sign is obsolete now and not used because it is unreliable in predicting a DVT.

The nurse is caring for an adolescent with hemophilia who reports pain and joint bleeding after playing baseball. The nurse should prioritize the client's

bleeding.

The nurse is assessing a client diagnosed with necrotizing pancreatitis. Which of the following assessment findings would be expected?

ecchymotic discoloration in the periumbilical region Choice A is correct. The discoloration of the abdomen and periumbilical area is known as Cullen's sign and indicates pancreatitis when it occurs in conjunction with other symptoms. This physical finding suggests necrotizing pancreatitis when pancreatic enzymes leak out of the pancreas causing damage to the surrounding tissue. Choices B, C, and D are incorrect. In pancreatitis, bowel sounds are generally diminished or absent. This is because peristalsis slows, causing the client to develop an ileus. Urinary manifestations such as dysuria and hematuria are not expected with pancreatitis.

The nurse locates the client's vastus lateralis by injecting

into the middle third of the anterolateral aspect of the thigh at 90 degrees The appropriate anatomical landmark to administer an intramuscular (IM) injection into the client's vastus lateralis is locating the thigh's anterolateral aspect. It extends in an adult from a handbreadth above the knee to a handbreadth below the greater trochanter of the femur. The nurse should use the middle third for the injection. To help relax the muscle, ask the client to lie flat with the knee slightly flexed and foot externally rotated or to assume a sitting position. IM injections are administered at 90 degrees. To administer an IM in the client's deltoid, it is located three finger widths below the acromion process. The landmark for an IM administered ventrogluteal is just below the iliac crest on the side of the thigh.

The client is at highest risk for developing

pernicious anemia The client is at highest risk for developing pernicious anemia because of the reduction of intrinsic factor. Instric factor is essential in the absorption of cobalamin. The client's neurological manifestations and physical examination findings of the oral ulcers are consistent with classic pernicious anemia.

The nurse is highly concerned about the client's risk for infection based on the client's vital signs

✓ Chemotherapy may cause an array of effects, including pancytopenia (low red blood cells; low platelets; low white blood cells) ✓ The nurse is responsible for monitoring the client's laboratory data and relaying manifestations of infection to the physician ✓ The nurse should educate and reinforce to the client risk reduction strategies for infection, such as the need for meticulous hand hygiene, wearing a surgical mask in public places, and maintaining good oral hygiene ✓ Neuropathic pain may consequently develop from medications such as doxorubicin which may be mitigated with prescribed gabapentin or pregabalin

The nurse is caring for a client receiving mechanical ventilation with positive end-expiratory pressure (PEEP). The nurse should monitor the client for which complication?

✓ PEEP is a setting that may be added to a mechanical ventilator, CPAP, or BiPAP ✓ PEEP is commonly prescribed for clients with acute respiratory distress syndrome (ARDS) because PEEP prevents alveolar collapse, allowing for better gas exchange, thus, improving oxygenation ✓ By improving gas exchange, therapeutically, the client will enjoy increased oxygen and less lactic acid from the stress of breathing (clients with low pulmonary compliance will have an increase in their breathing, thus, creating lactate and sending the client into acidosis ✓ PEEP can cause decreased venous return and lower the mean arterial pressure ✓ The blood pressure should be monitored closely for a client receiving PEEP because of the risk of hypotension ✓ PEEP also raises the client's risk for a stress ulcer ✓ 5-15 cm H2O is the range for PEEP that may be adjusted

The nurse is teaching a client about isoniazid (INH). Which of the following statements should the nurse include?

"A B-complex vitamin should be taken to help with the neuropathy." INH is a first-line therapy treatment for pulmonary tuberculosis. The major adverse effect associated with INH is peripheral neuropathy. This may be ameliorated by a client taking prescribed B-complex vitamins as INH depletes the stores of pyridoxine (Vitamin B6). Choices A, B, and D are incorrect. Reddish/orange secretions are a major effect seen with rifampin - not isoniazid. Yellowing of the eyes is not a normal side-effect as this is an indication of serious hepatoxicity. Antitubercular medications need to be taken for a prolonged period (at least six months). ✓ INH is the first-line therapy for pulmonary tuberculosis. ✓ This agent is often combined with another antitubercular medication because of emerging drug resistance. ✓ Hepatotoxicity is the most common adverse effect of most antitubercular drugs. ✓ The client should immediately report signs of hepatotoxicity, such as jaundice and clay-colored stools. ✓ INH may cause a client to develop peripheral neuropathies. Pyroxidine (vitamin B6) is commonly prescribed to prevent this occurrence.

A 12-year-old client with chronic asthma exacerbations has decided to try guided imagery as a way to manage the anxiety that is contributing to frequent asthma attacks. Which statement by the client indicates an understanding of this stress-reduction technique?

"I can do this anytime and anywhere when I feel anxious." Choice A is correct. Guided imagery is a stress-reduction technique that can be done in any place at any time. In fact, this is one of the biggest advantages of this technique. Anytime the patient begins to feel anxious, they can practice guided imagery. Choice B is incorrect. Guided imagery can be done in any position that the patient is most comfortable in. They do not have to by lying down unless they choose to. Choice C is incorrect. It is not necessary for the client's mom or anyone else to be present for guided imagery unless they choose so. Any person, or no one at all, can be present depending on the client's preferences. Choice D is incorrect. Music can but does not have to be played during guided imagery, again it depends on the client's preferences.

The nurse is discussing pernicious anemia with the client

"I may require additional doses of this medication." Vitamin B12 treatment for pernicious anemia secondary to gastric surgery may require indefinite treatment. Vitamin B12 is water-soluble and thus is readily excreted from the body. Repeated treatments are expected until the B12 levels normalize. Gastrointestinal manifestations (constipation, black tarry stools) are side effects associated with iron. Vitamin K is essential in blood clotting, and when Vitamin K is given, it assists with blood coagulation. Classic manifestations of vitamin B12 anemia include cognitive slowing, numbness and tingling in the extremities, glossitis, insomnia, and irritability ✓ This macrocytic anemia caused by a gastrectomy is treated with parenteral vitamin B12 injections (IM or SubQ) ✓ Common causes of pernicious anemia include decreased intake of vitamin B12 (eg, reduced intake of animal products, strict vegan diet, breastfeeding by a vitamin B12-deficient mother), decreased absorption of vitamin B12 (eg, gastrectomy, bariatric surgery, Crohn's disease, celiac disease, pancreatic insufficiency, bacterial overgrowth, fish tapeworm infection, gastric atrophy associated with aging), and medications such as metformin

The nurse is explaining the different aspects of ostomy care to a client with a newly created ileostomy. Which statement from the client indicates an understanding of the nurse's teaching?

"I need to cut the pouch to fit the stoma, allowing one-sixteenth of an inch of room around it." Choice A is correct. This is an accurate statement by the client. The client should be taught how to attach the pouch properly onto the stoma. The pouch should allow only 1/16 to 1/8 of an inch of room around the stoma. The client needs to understand that if the bag does not fit well, it can cause skin breakdown from contact with feces while allowing for passage of effluent through the stoma. Choice B is incorrect. The client must be encouraged to eat spinach, parsley, and yogurt, as these foods reduce drainage odor. Choice C is incorrect. The client should drink at least 2000 mL of water daily to prevent severe fluid and electrolyte imbalance as well as urolithiasis. Choice D is incorrect. It is not advisable to eat a large meal close to bedtime as ingested food passes through the ileostomy within 4-6 hours.

The nurse is caring for a client who has been prescribed depot medroxyprogesterone acetate. Which of the following statements, if made by the client, requires follow-up?

"I will need another injection in 8 weeks." Choice A is correct. Depot medroxyprogesterone acetate is an injection that provides contraception for 13 weeks. The client should return for another injection at 13-week intervals - not 8 weeks. ✓ Depot medroxyprogesterone acetate is an effective contraceptive given intramuscularly or subcutaneously every 13 weeks. ✓ While a client takes depot medroxyprogesterone acetate, calcium and vitamin D supplementation are recommended, coupled with weight-bearing exercises. ✓ Women with a high risk for cardiovascular disease and a stroke should not take depot medroxyprogesterone acetate. This medication may increase the risk for major adverse cardiovascular events (MACE).

A client has just been diagnosed with a terminal illness. She decides to execute a living will in the unit and asks the nurse to be the witness of the will. What is the most appropriate response by the nurse?

"I'm sorry, but under the law, we're not allowed to witness living wills." Choice A is correct. Nurses and other healthcare workers in the facility where the patient is receiving care are forbidden by law from becoming witnesses. Choice B is incorrect. This statement is inaccurate. Nurses and other healthcare workers in the facility where the patient is receiving care are forbidden by law from becoming witnesses. Choice C is incorrect. This statement is false. Witnesses for the signing of the will can be specific individuals; it does not necessarily mean only family. Choice D is incorrect. The hospital lawyer is not needed to be present in signing the living will.

A nurse is reviewing instructions with a client for the client's upcoming exercise electrocardiography (ECG). All of the following are appropriate statements by the nurse, except for which?

"You may have a light breakfast and a small cup of coffee before the exam." Choice B is correct. This statement by the nurse is incorrect and, therefore, the correct answer to the question. Typically, clients scheduled to undergo an exercise electrocardiography (ECG) (i.e., exercise stress test) are instructed to remain NPO for 4 to 6 hours before the stress test. Additionally, caffeine intake by a client may cause an alteration in the procedure's findings. Choice A is incorrect. The client should be instructed to wear loose, comfortable clothing with non-slip supportive footwear. In addition to facilitating exercise, stress test technicians will require access to the client's chest wall and all four extremities numerous times throughout the examination. Loose clothing will assist in facilitating access to those areas. Choice C is incorrect. The client will have continuous monitoring of their heart rate and rhythm (i.e., electrocardiogram) during the examination. A blood pressure cuff will also be continuously worn by the client during the testing, with readings taken at specific intervals. Choice D is incorrect. The client should be instructed to be well rested for the examination, refraining from strenuous activities the day and evening before the testing.

A woman comes into the emergency department complaining of insomnia, anxiety, difficulty breathing, and a sense of impending doom. After being assessed by the physician, no physiological abnormalities were found. However, the client is still anxious and apprehensive. What is the most appropriate statement from the nurse to the patient?

. "Can you think of anything that happened recently or in the past that might have triggered these feelings?" Choice B is correct. This is an appropriate response from the nurse because it offers reassurance to the client while providing an opportunity for the nurse to gain insight into the client's anxiety. Choice A is incorrect. This statement disregards the client's feelings and offers false reassurance. This is an inappropriate response by the nurse. Choice C is incorrect. Telling the client that you gave her some medication disregards her feelings and does not allow her to discuss those feelings. This statement also offers some form of false reassurance to the client. Choice D is incorrect. This statement disregards the client's feelings and offers false reassurance. This is an inappropriate response by the nurse.

The nurse is caring for a client with the following tracing on the electrocardiogram. When reviewing the client's medical history, which condition could be causing this dysrhythmia? See the image below.

. Graves' disease Choice A is correct. This tracing reflects sinus tachycardia (ST). ST can be caused by an array of conditions such as dehydration, hypo- and hyperglycemia, stress, anxiety, and thyroid conditions such as hyperthyroidism. Graves' disease is the most common cause of hyperthyroidism, and this increased metabolic and sympathetic activity would cause tachycardia. Choices B, C, and D are incorrect. Increased intracranial pressure would manifest with bradycardia. Hypothermia causes a slowing of metabolic and sympathetic activity; thus, bradycardia is a feature of this condition. Myxedema coma is an endocrine emergency marked by severe hypothyroidism. The hallmark of severe hypothyroidism is life-threatening bradycardia.

The nurse is conducting a staff in-service on managing an acute burn. The nurse should reinforce the utilization of which formula to guide fluid resuscitation?

4 mL x kg x TBSA burned The Parkland formula is widely used to calculate the fluid requirements following a major thermal burn. The formula of 4 mL x the client's weight in kilograms x the body surface area burned will determine the 24-hour fluid requirement. Once the total amount of fluid is calculated, divide it by two because of the two phases of fluid resuscitation (8 hours, then the remaining 16 hours). Choices B, C, and D are incorrect. 30 mL/kg is the formula used to determine repletion for a client with sepsis. 0.5 mL/kg/hr determines how much urine a client should produce every hour. UOP is monitored very closely during the acute phase of a burn to determine if the client is responding to treatment. 0.10 mL/kg/hr is not a formula used for any relevant medical calculation. The Parkland formula is a guide to determining the 24-hour fluid replacement following a major thermal burn. The phases of fluid replacement include the first eight hours and the remaining sixteen. The client's weight in kilograms will be multiplied by the total body surface area burned, then multiplied by 4 mL After the 24-hour fluid total is determined, it should be divided by two (for the two phases) The first eight hours begin at the time of injury - not hospitalization. Deduct any fluids given pre-hospital A central line is preferred to deliver this fluid Lactated Ringer's is commonly used as the fluid of choice because it may mitigate metabolic acidosis. Urine output is monitored closely to determine if the client is responding to treatment (0.5mL/kg/hr) The nurse should also monitor the mean arterial pressure (MAP) to determine if the client is responding (goal is > 65 mm Hg)

Which of the following clients does the nurse suspect would benefit most from placement of a nasogastric tube?

A 52-year-old client with leukemia who is receiving chemotherapy. Choice D is correct. The nurse suspects that a 52-year-old female with leukemia receiving chemotherapy would benefit most from a nasogastric tube. Nasogastric tubes are placed to help clients meet their nutritional needs. A client with leukemia has an increased need for calories and protein, but the chemotherapy treatment she is undergoing is likely to cause anorexia and nausea. This client could benefit from a nasogastric tube to help meet her nutritional needs.

Which client is at the highest risk for developing a decubitus ulcer among the following patients in a long-term care facility?

A 79-year-old malnourished client on bed rest. Choice C is correct. Prolonged inadequate nutrition causes weight loss, muscle atrophy, and the loss of subcutaneous tissue. These three conditions reduce the amount of padding between the skin and bones, thus increasing the risk of pressure ulcer development. Specifically, inadequate protein, carbohydrates, fluids, zinc, and vitamin C intake contribute to pressure ulcer formation. Immobility resulting from prolonged bed rest is a risk factor. Several factors contribute to the formation of pressure ulcers: friction and shearing, immobility, inadequate nutrition, fecal and urinary incontinence, decreased mental status, diminished sensation, excessive body heat, advanced age, and certain chronic conditions. Choice A is incorrect. Stool incontinence can be a source of skin irritation and contribute to pressure ulcer formation. However, it is not the highest risk for developing a decubitus ulcer. A malnourished client on bed rest is at the highest risk. Choice B is incorrect. While diabetics may develop skin ulcers and experience slower healing times, ambulatory patients are not at risk for developing pressure sores. Choice D is incorrect. As long as the client is ambulating, occasional wheelchair use and obesity are not significant risk factors for developing a decubitus ulcer.

The nurse is caring for a client who was prescribed a clear liquid diet. Which dietary items would be appropriate for the nurse to include? Select all that apply.

A clear liquid diet is usually transparent (to light) dietary items that do not contain dairy or pulp. Items such as water, gelatin, fat-free bouillon, hot tea, apple juice, seltzer, lemonade, and ginger ale are acceptable. Clear hard candy is acceptable because it is a clear liquid when melted. Salt and sugar are food additives that are permitted. Incorrect responses A full liquid is the next step when the diet is advanced. This diet contains opaque liquids. A full-liquid diet usually contains pulp and dairy. For example, coffee is a clear liquid, whereas a coffee with creamer or milk is a full liquid. Items that are full liquid include sherbert, milkshakes, frozen yogurt, pudding, strained soups, and coffee with dairy (or nondairy alternatives such as oat milk). ✓ A clear liquid diet is easily absorbed and digested ✓ This diet often is used for a PO (by mouth) challenge if the client is experiencing nausea and vomiting, if they can tolerate it without further vomiting it is considered a successful challenge ✓ The disadvantage of a clear liquid diet is that it provides very little nutritional value ✓ Salt and sugar additives are permitted

The nurse is educating a group of nursing students about fetal alcohol spectrum disorders (FASDs). Which of the following features are characteristic of FASDs?

A is correct. Intellectual disabilities are characteristic features of FASDs. Prenatal alcohol exposure can cause neurodevelopmental issues, leading to intellectual disabilities and cognitive impairments in children. B is correct. Another characteristic feature of FASDs is facial abnormalities. Children affected by prenatal alcohol exposure may have distinctive facial features, including a smooth philtrum (the area between the upper lip and nose), a thin upper lip, and small eye openings. C is correct. Growth delays are often seen in individuals with FASDs. Prenatal alcohol exposure can affect overall growth, leading to below-average height and weight in affected individuals. D is correct. Motor impairments, such as poor coordination and balance difficulties, can occur in individuals with FASDs. These difficulties may affect fine motor skills and gross motor functions. Choice E is incorrect. E is incorrect. While anxiety disorders are not typically recognized as a characteristic feature of FASDs, it is not directly associated with prenatal alcohol exposure. Therefore, this option should be excluded as it does not align with FASD characteristics.

A nurse on a rehabilitation floor is educating a post-cerebrovascular accident (CVA) client with right-sided hemiplegia on how to transfer himself from a bed to a wheelchair without assistance. Which of the following statements, if communicated by the client, would indicate the need for additional education on this transfer technique?

A. "I'm going to put the wheelchair on the right side of my bed and make sure it is locked." Choice A is correct. This statement by the client is incorrect and needs to be addressed by the nurse. Here, the client is affected by right-sided hemiplegia following his CVA. The client should always place the wheelchair on the unaffected side (here, his left side) to facilitate a safe and ergonomic transfer using the strength remaining in his left arm and leg. Placing the wheelchair on the right side of the bed (i.e., on the client's affected side) greatly increases the client's fall risk and risk for injury. Choice B is incorrect. This statement is correct and does not require further teaching. The client should use his unaffected hand and foot to maneuver towards the edge of the bed. Choice C is incorrect. This statement is correct and does not require additional instruction. After the client is positioned in a sitting position on the side of the bed, he should push himself up to a standing position with his unaffected leg, using his unaffected arm to grasp the locked wheelchair and gently sit down. Choice D is incorrect. This statement is correct and does not require additional teaching. Once the client is in a sitting position on the side of the bed, he should push himself up to a standing position with his unaffected leg, using his unaffected arm to grasp the locked wheelchair and gently sit down. When provided a list of steps necessary for a client affected with hemiplegia when transferring himself from a bed to a wheelchair without assistance, recognize that the locked wheelchair should be placed on the client's unaffected side. For clients with hemiplegia, significant inpatient rehabilitation must occur prior to discharge. When performing client education for hemiplegia clients transferring from a bed to a wheelchair, emphasize the need for the wheelchair to be locked and placed on the client's unaffected side. Many clients utilize a slide board to facilitate the transfer from the bed to a wheelchair.

A parent in a pediatric clinic asks the nurse when the soft spots on their baby's head will harden. The most appropriate response by the nurse would be:

A. "These soft spots are called fontanels. Typically, the one towards the front of the head closes between 12 and 18 months, and the one on the back of the head closes by two months old." Choice A is correct. Fontanels facilitate the bony plates of the baby's skull to flex and allow the baby's head through the birth canal. Typically, the posterior fontanel closes by two months of age, while the anterior fontanel fuses between 12 and 18 months. Six fontanels (also commonly referred to as fontanelles) are present during infancy, the most notable being the anterior and posterior fontanels. Certain conditions, such as dehydration or infection, can alter the appearance of the fontanelles, causing them to sink or bulge, respectively. Parents and caregivers should be educated to contact their child's health care provider (HCP) if any changes are noted in the appearance of any of their child's fontanels. The primary cause of a sunken fontanel is due to dehydration. Conversely, a bulging fontanel occurs due to a rise in intracranial pressure, typically indicative of one or more of the following pathologies: hydrocephalus, hypoxemia, meningitis, trauma, or hemorrhage (of note, this is not an all-inclusive list).

The nurse is caring for a client who sustained an electrical burn. Which priority action should the nurse take?

A. Obtain an electrocardiogram (ECG) Choice A is correct. Electrical burns are serious and require the client to undergo cardiac monitoring because of the risk of dysrhythmias. The nurse's priority action is to obtain telemetry monitoring or perform a 12-lead electrocardiogram. Choices B, C, and D are incorrect. These types of burns cause an 'iceberg' effect where the client's external injuries appear minor, but the internal injuries may be catastrophic. Electrical burns may trigger immediate ventricular fibrillation or asystole. Thus, it is important to monitor the client's cardiovascular status immediately following this type of injury.

The nurse is caring for an infant with developmental dysplasia of the hip (DDH). Which of the following prescriptions would the nurse anticipate from the primary healthcare provider (PHCP)?

A. Pavlik harness Choice A is correct. The Pavlik harness is utilized for the treatment of DDH. The goal of the therapy is to keep the hips abducted as much as possible. Choices B, C, and D are incorrect. Compression hose, knee immobilizer, and continuous passive motion are not utilized in the treatment of DDH. The goal is to promote immobilization of the hip. Therefore, the continuous passive motion may be unhelpful.

Which of the following statements correctly outlines the proper flow of blood through the heart?

A. Superior and Inferior vena cavas → Right atrium → Tricuspid valve → Right ventricle → Pulmonary valve → Pulmonary artery → Lungs → Pulmonary veins → Left atrium → Mitral valve → Left ventricle → Aortic valve → Aorta → Systemic circulation

The nurse is caring for a child who is receiving prescribed methylphenidate. Which of the following findings should be reported to the primary healthcare provider (PHCP)?

A. Weight loss of 3 kilograms Choice A is correct. Children receiving psychostimulant medications such as methylphenidate should be monitored closely for weight loss which may inhibit their ability to meet their growth milestones. This finding of a three-kilogram weight loss is necessary to report to the PHCP. Choices B, C, and D are incorrect. Dry mouth, occasional headaches, and insomnia are common side effects associated with methylphenidate. These findings are concerning but may be mitigated through other interventions. The weight loss is quite concerning as this threatens the child's physiological health. Methylphenidate is a psychostimulant effective in treating attention deficit hyperactivity disorder (ADHD). This medication may be used, or an amphetamine may be utilized to treat this disorder. While non-stimulant medications are available, psychostimulant medications may produce rapid results. Adversely, a child's growth pattern needs to be monitored closely as this medication may cause anorexia and weight loss.

The nurse is caring for assigned clients. The nurse should initially follow up on the client who is

A. three days postoperative following transsphenoidal hypophysectomy and has a temperature of 101°F (38.3°C). Choice A is correct. A complication of transsphenoidal hypophysectomy is meningitis. The client needs to be immediately assessed for other manifestations of meningitis, including photophobia, nuchal rigidity, and altered mentation. Complications following this surgery include CSF leakage, infection, optic nerve damage, and diabetes insipidus. Choices B, C, and D are incorrect. These findings do not require follow-up because all of these situations are expected. Pneumothorax produces diminished to absent breath sounds on the affected side because of lung collapse. Thus, the treatment of the chest tube is to increase negative pressure in the pleural space to promote expansion. Albuterol causes a discharge of the body's epinephrine, thus causing the client to feel nervous or jittery. This, along with an elevation in the heart rate, is an expectation and will resolve a few hours after the treatment. Peritoneal dialysis is when the client instills hypertonic fluid into the peritoneum to draw out waste products. Cramping during fluid instillation is common and can be mitigated by warming the solution and slowing down fluid instillation. ✓ Hypophysectomy is a surgery to remove a pituitary tumor or remove the pituitary gland ✓ The approach is most likely transnasal via endoscopy ✓ Complications of this surgical procedure include CSF leak, infection, diabetes insipidus, and hypopituitarism ✓ Postoperatively, the head of the bed should be elevated semi- to high-Fowler's ✓ The client should be instructed not to cough, sneeze, or blow their nose ✓ The client should not bend at their waist to prevent the rising of intracranial pressure ✓ The client should use mouthwash or dental floss for several days to allow the surgical incision to heal ✓ Fever and photophobia should be reported because this is a concerning finding for postoperative meningitis ✓ Drainage should be monitored for CSF which would be a light yellow color at the edge of the clear drainage on the dressing is called the halo sign and indicates CSF

The nurse is caring for a client with angle-closure glaucoma. Which prescription should the nurse anticipate from the primary healthcare provider (PHCP)?

Acetazolamide Choice A is correct. Acetazolamide is a diuretic given intravenously or orally to a client with angle-closure glaucoma. This medication causes a reduction of aqueous humor, which is helpful in the management of angle-closure glaucoma that is marked by an IOP greater than 30 mmHg. Angle-closure glaucoma is a medical emergency that may cause blindness if not promptly treated. Choices B, C, and D are incorrect. Anticholinergics, tricyclic antidepressants, decongestants, and antihistamines should be avoided because these medications raise the IOP. Glaucoma is the leading cause of blindness in North America, and the client's medications must get reviewed so the client avoids taking these types of drugs. ✓ Angle-closure glaucoma is an ocular emergency that occurs when the intraocular pressure exceeds 30 mmHg (normal is 10-21 mmHg). ✓ The client may experience manifestations such as ipsilateral headache, brow pain, nausea, and blurred vision. ✓ Emergent prescriptions such as timolol eye drops are used to lower intraocular pressure. ✓ Other useful agents include intraocular timolol. ✓ The client should be placed supine, which will assist in the lens falling away from the iris, decreasing the pupillary block.

The nurse is performing a physical assessment on a client with Cushing's disease. Which assessment findings should the nurse expect? Select all that apply.

Acne Hirsutism Buffalo hump Truncal obesity Choices B, C, D, and E are correct. Acne is an expected symptom of Cushing's disease. This is due to increased sex hormones, such as testosterone, estrogen, and progesterone. Excessive levels of these hormones cause oily skin to build up, often leading to acne. Hirsutism is defined as excessive body hair in either men or women in places where the hair usually is absent, such as the chin or cheeks of the face. Hirsutism is caused by increased androgens, or male sex hormones such as testosterone, in the body. A buffalo hump is the classic sign of Cushing's disease. This refers to a lump of fat that develops between the shoulder blades on the top of the back. It is due to the excessive amount of glucocorticoids that clients with Cushing's disease have. Glucocorticoids cause the breakdown of fats; when there are too many, they can cause fat redistribution. This often leads to fat in odd places, such as a buffalo hump. This excessive fat storage causes an individual to develop truncal obesity. Choice A is incorrect. Hyper- not hypotension, would be expected in a client with Cushing's. This is due to too many mineralocorticoids, specifically aldosterone. With increased aldosterone levels, the body retains too much sodium and water. With increased fluid in the vasculature, the blood pressure rises, and the client is hypertensive. Hypotension would be expected in a client with Addison's disease, whose body has a decreased amount of steroids.

You are providing discharge teaching for a 3-year-old patient with CHF who has been prescribed digoxin. Which of the following medication instructions should be included in the discharge teaching?

Administer digoxin one hour before or two hours after meals. Call the doctor if the child starts eating poorly and vomiting frequently. Administer at same time every day ✓ Digoxin is usually given orally, and the child should be instructed to swallow the medication rather than chewing or crushing it. If a child has difficulty swallowing the medication, the healthcare provider may consider using an alternative form of administration, such as a liquid or crushed tablet mixed with food. ✓ Children taking digoxin should be monitored for signs of toxicity, which can include nausea, vomiting, headache, visual changes, or irregular heart rhythm. Healthcare providers should also monitor the child's heart rate, rhythm, and electrolyte levels, which can affect the effectiveness and safety of digoxin. ✓ Parents and caregivers should be educated on the proper administration of digoxin, including the importance of giving the medication at the same time each day and not missing doses. They should also be informed of the signs and symptoms of toxicity and instructed to contact the healthcare provider if these occur.

Which of the following healthcare providers are responsible for documenting care provided to a patient?

All staff members should document all of the care that they have provided. Choice C is correct. All staff members, including unlicensed assistive staff like nursing assistants, document and sign all of the care that they have personally provided. For example, the nursing assistants will document the vital signs that they have taken; the licensed practical nurses will document all of the treatments and medications that they have given to the patient, and the registered nurse will document nursing diagnoses and assessments that they have completed. There is an old saying among healthcare professionals that have been passed on to new generations. The saying is, "I don't care what you did; if you didn't document it, you didn't do it." Documentation is an essential part of patient care. A patient's complete medical record is a legal document. Proper documentation means 1. The person who provided care should document what care/treatment/medication was given and how the patient responded. 2. If care is delegated to another person, it should be noted to whom the responsibility was assigned; proper documentation AND follow-up should be done. Choice A is incorrect. Each person providing care should personally document the attention that he/she provided. Choice B is incorrect. Although the RN or charge nurse is responsible for making sure tasks are delegated to the appropriate personnel, only the person who performs the care should document the care that was provided. Choice D is incorrect. The person providing care should document the care followed by his/her signature.

For each client finding below, click to specify if the finding is consistent with the disease process of benzodiazepine toxicity, bacterial meningitis, or influenza

Altered mental status is consistent with benzodiazepine toxicity and bacterial meningitis. Benzodiazepines are a central nervous depressant and may cause ataxia, decreased responsiveness, slurred speech, and decreased purposeful movements. Bacterial meningitis is concerning because it causes the classic triad of symptoms fever, photophobia, and nuchal rigidity. The client also will experience altered mentation due to irritation to the meninges. Fever is a classic finding in both influenza and bacterial meningitis. Fever is not associated with benzodiazepine toxicity. This would be found with cocaine toxicity since cocaine is a CNS stimulant. Petechial rash may occur with bacterial meningitis, which may be found on any body region. This type of rash is caused by disrupted capillaries caused by coagulopathy or other hematological abnormality. Altered speech pattern is found in this client as she is mumbling. This is expected with clients with meningitis as the CNS alteration comes from irritation to the meninges. The client reported neck stiffness before bed, which may be inferred as nuchal rigidity, a part of the classic triad of bacterial meningitis. Influenza causes muscle aches but not neck stiffness. The nurse should implement droplet transmission-based precautions for a client with suspected bacterial meningitis. Standard Precautions are appropriate for all patients with meningitis unless the patient has a bacterial type transmitted by droplets, such as N. meningitides and H. influenza. Since the exact organism is unknown, the nurse should always isolate suspected cases until confirmation is obtained. Droplet transmission-based precautions do not require the door to be kept closed. Droplet transmission-based precautions require staff and visitors in the room to wear a surgical mask (not a face shield) upon entry into the client's room. A quiet environment is appropriate to avoid raising the client's intracranial pressure, which may occur with bacterial meningitis. This includes keeping the television off and limiting visitors. The nurse should perform frequent neurological assessments to determine if the client's mental status is improving or declining. At a minimum, the neurological assessment should occur every two hours. The client will need an immediate collection of blood cultures and laboratory work. Additionally, the client will require medication to break the tachycardia and fever, including fluids and antipyretics. Thus, establishing a peripheral vascular access device is essential. An immediate CT scan of the head and a lumbar puncture is essential to establish the diagnosis of bacterial meningitis. An EEG has no relevance in managing this condition as this testing is performed for individuals who may have experienced a seizure. A lumbar puncture is critical to diagnosing bacterial meningitis. The LP is performed with the client positioned in the lateral recumbent position with the knees up to the abdomen and the chin onto the chest. A strict aseptic technique is necessary for the individual performing the LP, and the nurse should immediately send the laboratory specimens off for processing. Before an LP, laboratory work, specifically, the platelet count and clotting factors (PT/INR), are necessary because a potential contraindication would be if the client had thrombocytopenia which could cause bleeding and swelling leading to neurological impairment. The client should be positioned completely flat after the procedure, not before. Contrast dye is not involved in this procedure. Ketorolac is a nonsteroidal anti-inflammatory drug used for pain or lower fever. The nurse should monitor the client's temperature to assess the efficacy of the NSAID, with the treatment goal being to lower the temperature. Ketorolac is not a diuretic, and ketorolac does not adversely cause hypoglycemia. Ceftriaxone is an antibiotic essential in the treatment of bacterial meningitis. This medication should be administered immediately after cultures are collected, and the LP is performed. Delaying the antibiotic administration could have serious negative effects on the client. Ceftriaxone should be administered after the cultures are obtained to collect an accurate specimen. Finally, ceftriaxone has no cross-sensitivity with doxycycline. Ceftriaxone would potentially have a minimal cross-sensitivity if the client were allergic to penicillin. Seizure precautions are standard when caring for a client with bacterial meningitis because of the seizure risk. Padding the side rails of the bed - not the bed frame is helpful in preventing a head injury. A key component of seizure precautions is to have patent vascular access, so a prescribed medication, like lorazepam, may be administered to break the seizure. Additional lighting in the room is not a component of seizure precautions. This would be helpful in fall precautions.

The nurse is caring for a client on bed rest for a week following a right hip fracture. Which of the following findings, if noted in the client, would indicate signs of complications due to immobility? Select all that apply.

An area of the client's sacrum is unable to be blanched Crackles in the bases of the client's lungs Swelling and tenderness in the left calf Choices A, C, and D are correct. A patient on prolonged bedrest will experience complications such as decubitus ulcers (bedsores), atelectasis, and deep vein thrombosis unless preventive measures are deployed. The nurse should be aware of the signs suggestive of these complications. (Choice A) A localized area of redness (usually over a bony prominence) that does not blanch (does not turn white when pressed with a finger) and with intact skin suggests a Stage I decubitus ulcer/ pressure sore. Sacral decubitus or a pressure ulcer is a common complication with immobility. (Choice C) Crackles in an immobilized patient suggest atelectasis. The presence of crackles in bilateral lung bases indicates bibasilar atelectasis. Atelectasis refers to the collapse of the lung's alveoli (tiny air sacs). In a patient confined to prolonged bed rest, the mucus pools in the lower portions of the airway. Additionally, the diameter of bronchioles decreases in the supine position. These two factors, pooled mucus, and reduced airway size, make it difficult to clear the secretions. The supine position also places weight over the ribcage and makes the breathing labored. Therefore, the patient takes fewer breaths and is not deep enough. All these factors result in a collapse of the small airways and alveoli, leading to atelectasis. (Choice D) Swelling and tenderness in the calf suggest deep vein thrombosis (DVT). In an immobilized patient, venous stasis occurs, predisposing to venous thrombosis. DVT and pulmonary embolism are some of the severe complications of immobility. Choice B is incorrect. Scleral icterus and yellow skin suggest jaundice. Hepatitis from various causes (medications, viral infections, toxins) may result in jaundice. Liver function tests are not affected by immobility. Choice E is incorrect. Urinary incontinence is a complication of immobility. However, a patient using a bedpan to void is the desired goal and is not a complication of immobility.

A patient tells the nurse that she is ashamed of how her hair looks and wants to wash her hair before her daily tests and appointments are performed. How should the nurse prioritize the patient's care?

Arrange to wash the patient's hair first, perform hygiene, and then schedule diagnostic testing and counseling. Choice D is correct. As long as time constraints permit, the most immediate priorities when scheduling nursing care are the priorities identified by the patient as being the most important. Choices A, B, and C are incorrect. In this case, washing the patient's hair and assisting with hygiene puts the patient first and sets the tone for a productive nurse-patient relationship. Diagnostic tests and dressing changes in a stable patient can certainly wait until after the patient-identified priorities are addressed.

The nurse is caring for a client who is six hours post-partum. The client informs the nurse that they have changed their peri-pad four times in the last six hours. The nurse should take which action?

Assess the client for hemorrhage Choice C is correct. The initial action is to assess the client for postpartum hemorrhage. The normal pad count after birth is one pad every two hours. If the client should exceed this, then the client should be assessed for postpartum hemorrhage. Choices A, B, and D are incorrect. Documenting the findings as expected would be inappropriate. This client needs to be assessed for potential postpartum hemorrhage. Massaging the client's fundus and encouraging the client to void would be appropriate measures if the hemorrhage is confirmed by the nurse. This can only come through assessment. Postpartum hemorrhage (PPH) is serious and a significant contributor to maternal death in morbidity worldwide Risk factors and causes of PPH include Multiple gestation Uterine atony Macrosomia birth (increased risk of lacerations) Hydramnios (large amniotic fluid volume making uterine contraction difficult) Retained placenta (it will now allow the uterus to fully contract) Manual removal of the placenta Clotting disorders Lacerations Any delivery that was assisted with a tool or instrument (increased risk of lacerations) Manifestations of a client experiencing PPH include excessive lochia, uterine tenderness, and unstable vital signs (tachycardia, hypotension) Treatment includes prompt recognition and activation of a PPH protocol (each facility is required to have a protocol that streamlines treatment). If uterine atony is the cause, a fundal massage is necessary. Blood product replacement, intravenous oxytocin, and/or intramuscular (IM) methylergonovine.

The nurse enters a client's room who is found on the ground. The nurse should perform which initial action?

Assess the client's level of consciousness Choice A is correct. For a client found down on the ground, the nurse should immediately implement basic life support measures, including initially assessing the client's level of consciousness. If the client is unconscious, the nurse should stay with the client and shout for help. Choices B, C, and D are incorrect. Assessing the client's level of consciousness is the priority over assessing the client for injuries. Examining the client for injuries is a secondary assessment as their immediate concern is their overall stability. The RRT may need to be activated depending on the outcome of the client's level of consciousness. According to the BLS algorithm, the client's level of consciousness should be prioritized over the assessment of the carotid pulse.

The nurse is planning a staff development conference about the care of transgender clients. Which of the following information should the nurse include?

At the start of the interview, inquire about the client's preferred pronoun. Transgender individuals feel a variance between gender and natal sex. Clients who are transgender may be reluctant to seek healthcare. Inquire about any current or future plans for hormone therapy.

The nurse works in a long-term psychiatric rehabilitation center and is assigned to a patient with debilitating agoraphobia. The patient is going through desensitization therapy. Which of the following interventions is an appropriate part of this treatment?

B is correct. Since the client has agoraphobia, they fear leaving places where they feel safe and comfortable. Desensitization to this fear should occur gradually, starting with small steps outside their comfort zone. Taking a short walk in the hallway outside their room, where they feel safe, is an appropriate choice for desensitization therapy. Choice C is correct. Building rapport and trust with the client is crucial during desensitization therapy. Trust enables the client to feel comfortable and supported, allowing for successful progress as they are guided to face their fears gradually. Choice E is correct. Agoraphobia is classified as an anxiety disorder. Teaching and encouraging relaxation techniques, such as deep breathing exercises, progressive muscle relaxation, and mindfulness techniques, would be appropriate to help the client manage anxiety symptoms. Choices A and D are incorrect. A is incorrect. When treating clients with phobias, focusing solely on discussing what frequently triggers the phobia is not recommended. This approach can increase anxiety and keep clients fixated on their fears, potentially triggering panic attacks. D is incorrect. Desensitization therapy involves gradual exposure to feared situations rather than immediately confronting the most challenging aspects of the phobia. Suggesting a drastic leap could overwhelm the client and potentially lead to setbacks in their progress. Starting small steps and gradually working towards bigger goals is more effective in desensitization therapy.

The nurse is caring for a client experiencing acute mountain sickness (AMS). The nurse anticipates a prescription for which medication?

B. Acetazolamide Acetazolamide, a carbonic anhydrase inhibitor, is commonly prescribed to prevent or treat AMS. It acts by causing a bicarbonate diuresis, which rids the body of excess fluid and induces metabolic acidosis. The acidotic state increases the respiratory rate and decreases the occurrence of periodic respiration during sleep at night. In this way, it helps clients acclimate faster to a high altitude. By increasing the client's respiratory rate, the client can perfuse more oxygen. It is preferred that this medication be taken 24 hours prior to the ascent. Acute mountain syndrome (AMS) causes increased sympathetic nervous system activity, increased heart rate, blood pressure, and cardiac output. Pulmonary artery pressure rises as an effect of generalized hypoxia-induced pulmonary vasoconstriction. Cerebral blood flow increases to maintain cerebral oxygen delivery. All of these processes call for an increased need for oxygen. Treatment for AMS includes supplemental oxygen (if available) and getting the client to a lower altitude. The client may also benefit from prescribed acetazolamide. Sodium bicarbonate is not indicated for AMS. The goal is to create metabolic acidosis by giving the client prescribed acetazolamide, increasing the client's respiratory rate, and increasing oxygen delivery. Tamsulosin and dutasteride are indicated in benign prostatic hyperplasia (BPH) treatment.

While training a new RN in the emergency department, the nurse attends to a client with Guillain-Barre Syndrome. The new RN asks what may have caused this condition. Which of the following occurrences in the patient's history is most likely a contributing factor?

B. An upper respiratory infection about a month ago Choice B is correct. Upper respiratory infections or stomach infections correlate with the development of Guillain-Barre syndrome. Guillain-Barre syndrome is a disorder that involves the peripheral and cranial nerves causing ascending paralysis. Choice A is incorrect. Spinal cord injuries are not generally associated with Guillain-Barre syndrome. Choice C is incorrect. Hydrocephaly, as an infant, is not associated with Guillain-Barre syndrome. Hydrocephaly refers to the enlargement of the ventricles. Choice D is incorrect. Joint injuries are not associated with Guillain-Barre syndrome.

A 55-year old female has a complete knee replacement on her left knee. Which symptom would be the most likely to indicate a severe adverse reaction after surgery?

B. Capillary refill 5+ for the left foot Choice B is correct. Assessments after knee surgery should include the 5 P's: pain, pallor, pulse, paralysis, and paresthesia. Capillary refill should be assessed during the pallor assessment. A normal capillary refill is less than two seconds. The capillary refill, in this case, is 5 seconds, which is indicative of a problem. Choice A is incorrect. This patient may not be able to move her leg due to pain after the surgery. Choice C is incorrect. This patient will have pain at the surgery incision site. If the patient continues to have severe and uncontrollable pain in the entire leg, the nurse should worry about compartment syndrome. Choice D is incorrect. This is a normal finding.

You are called to assess a 4-year-old patient who has suffered second and third-degree burns to her chest, abdomen, and legs. It is estimated that about 40% of her TBSA is burned. Upon assessment, her vital signs are as follows: HR: 140 RR: 44 BP: 90/60 SpO2: 88% on Room Air Which of the following interventions is the priority?

B. Intubation and mechanical ventilation Choice B is correct. Intubation and mechanical ventilation are the priority for this patient. Intubation is the A in the ABC's mnemonic and stands for airway. The stem of the question states that this patient has burns to her chest. You know that smoke inhalation can burn the trachea and compromise the airway, and wounds to the chest are an indication that inhalation injury has likely occurred. Additionally, the patient is tachycardic, tachypneic, and desaturated. She is working hard to try to compensate by increasing her heart rate and respiratory rate. Still, it is not keeping up with her oxygenation and perfusion needs, as evidenced by her desaturation in room air. This patient needs intubation and mechanical ventilation to secure an airway and prevent rapid respiratory failure due to inhalation injury. Remember your ABCs! Airway, Breathing, Circulation. This is the correct order of priority actions! You must address the airway first! Choice A is incorrect. Although fluid replacement will be necessary during the treatment of a burn victim, it is not the priority action and is, therefore, the wrong answer to this question. Every answer here is a correct action that would be necessary for the care of your patient, but the priority is not a fluid replacement. If you picked this answer, you might be remembering that a large amount of third spacing occurs in the first 24 hours after a burn. Hypovolemic shock can ensue if the adequate fluid replacement does not occur. This falls under the C in your ABCs and stands for circulation. While fluid replacement is essential, you must address the airway as your priority. Choice C is incorrect. While wound debridement is undoubtedly essential for infection prevention and healing in the burn victim, there is a more urgent priority. This is not a part of your ABCs and is not an immediate priority. Choice D is incorrect. It is correct to implement standard precautions for a burn victim to prevent infection, but there is a more urgent priority. This is not a part of your ABCs and is not an immediate priority.

A 68-year-old woman arrives at the emergency department after feeling dizzy. After assessing the patient, the nurse notices hypotension, muffled heart tones, and jugular venous distention. What does the nurse suspect that this patient has? What is this triad called?

B. Pericardial tamponade; Beck's triad Choice B is correct. Beck's triad is a symptom triad that indicates pericardial tamponade. Hypotension occurs because the patient is actively losing blood into the pericardial space. This sac can hold as little as 150 mL to 1,000 mL and impedes cardiac output. Jugular vein distention (JVD) occurs because the heart is compressed, which leads to delayed venous return. Blood pools in the veins and this can be assessed as jugular vein distention. Choice A is incorrect. Cushing's triad is associated with increased ICP and is characterized by irregular respirations, widened pulse pressure, and bradycardia. Choice C is incorrect. Increased ICP and Cushing's triad is not what this patient is suffering from. Choice D is incorrect. This patient is experiencing symptoms that are called Beck's triad. However, this triad is indicative of pericardial tamponade, not a pleural effusion. A pleural effusion is known as fluid in the pleural space of the lung.

Which statement about behavior management is accurate?

B. Skinner's theory scientifically supports behavior management techniques and procedures. Choice B is correct. According to Skinner's theory, changes in behavior result from an individual's response(s) to the specific events, or stimuli, which occur in their respective environment. According to Skinner's research, everything we do and are shaped by our experience of punishment and reward. More specifically, when a stimulus-response pattern is rewarded, the individual is conditioned to respond similarly in the future. The key to Skinner's theory is reinforcement, or, more specifically, anything that strengthens the desired response. The central tenet of Skinner's work is that positively reinforced behavior will reoccur. When an individual is rewarded for a specific behavior, that individual is more likely to repeat the behavior due to positive reinforcement. Conversely, under Skinner's theory, negative reinforcement involves removing an undesirable stimulus to increase a behavior (not to be confused with a "negative punishment," where one would remove a pleasant stimulus to decrease a behavior). Despite Skinner's operant conditioning findings dating back to publications from the 1960s, many of these scientific theories continue to be implemented today. While Skinner's research and findings greatly influence countless fields, education is one of the most notable. Education appears to have been affected by Skinner's theories at all educational levels, having been integrated into all aspects of classroom management by instructors, instructional designers, administrators, etc., continuing to this day. Skinner's theories relating to behavior management techniques and procedures continue to be cited and implemented today. Choice A is incorrect. Research has shown that negative feedback is often a valuable form of feedback, as it allows an individual (or client) the opportunity to scrutinize their position, role, or behavior while concurrently alerting the client to a significant change(s) that is often needed. Based on the minimal information provided within this answer selection, nothing within Choice A indicates a breach of ethics has occurred. Choice C is incorrect. According to Orem's nursing theory, self-care consists of activities in which clients engage to maintain, restore, or improve health. The fundamental philosophy of Orem's theory is that in addition to all clients wanting to care for themselves, clients are able to improve more quickly and thoroughly by performing their own care as much as possible. Orem's nursing theory is not associated with behavior management. Choice D is incorrect. Research has shown that negative feedback is often a valuable form of feedback, as it allows an individual (or client) the opportunity to scrutinize their position, role, or behavior while concurrently alerting the client to a significant change(s) that is often needed.

The risk manager reviews an incident report completed by a nurse regarding a client's fall. Which finding in the report demonstrates inappropriate documentation?

B. Subjective factors preceding the fall. Choice B is correct. The purpose of an incident report is to provide an objective account of an incident/occurrence, in order to identify issues with current practices, improve policies, and potentially investigate situations of negligence/malpractice. Subjectivity should be excluded from a report because subjectivity allows for opinions on details that may not be true (example, stating I believe the client fell because he did not follow instruction) would be inappropriate. Choices A, C, and D are incorrect. An incident report should include the client's account of the fall (stated in quotations). An incident report should include any injuries sustained or adverse effects noted as a result of the fall and the monitoring and assessment performed following the event. An incident report should include the names of any witnesses to the fall.

The nurse is reviewing eating disorders with a group of students. It would be correct for the nurse to identify that the primary differential between anorexia nervosa and bulimia nervosa is a client's

B. body mass index (BMI). Choice B is correct. Clients with anorexia nervosa often present with a body mass index (BMI) of < 18.5 (i.e., underweight). This is due to the severe caloric restriction anorexia nervosa clients implement on themselves. Conversely, the body weight of clients with bulimia nervosa tends to fluctuate around normal, periodically going above this range (i.e., the majority of bulimia nervosa clients have a BMI between 18.5 and 30). This higher BMI is due to the binging and purging of high-calorie foods by clients with bulimia nervosa. Binging and purging are clinical features of both bulimia and anorexia nervosa. It cannot be used to determine the client's condition. Choices A, C, and D are incorrect. Both bulimia nervosa and anorexia nervosa have significant overlaps in their clinical features. Erosion at a client's dentition is an indicator of binging and purging, which can be found in both bulimia and anorexia nervosa. A key feature of both bulimia nervosa and anorexia nervosa is the client's distortion of their body weight and shape. ✓ Significant overlap occurs between bulimia nervosa and anorexia nervosa ✓ Physically, the client with anorexia nervosa is significantly underweight with a BMI < 18.5 ✓ Both eating disorders can cause binging and purging ✓ Both eating disorders cause a client to have a distorted view of food and themselves ✓ Both eating disorders may have other psychiatric co-morbidities such as obsessive-compulsive personality disorder or depression ✓ Fluoxetine has proven efficacy in the treatment of bulimia nervosa

The nurse is caring for a client in labor. The following tracing was on the fetal heart rate monitoring strip. The nurse recognizes that this tracing is a

B. late deceleration. Choice B is correct. This strip indicates a late deceleration. Late decelerations are visually apparent and usually symmetric in shape, with a gradual decrease and return of the fetal heart rate (FHR) to baseline. Late decelerations are caused by decreased perfusion to the fetus. Maternal repositioning is an effective intervention for this nonreassuring pattern. Other interventions include oxygen administration and the administration of intravenous isotonic fluids. Choice A is incorrect. This strip does not show a variable fetal deceleration. This strip indicates the presence of another fetal heart pattern other than variable fetal deceleration. Choice C is incorrect. This strip does not show early fetal deceleration. This strip indicates the presence of another fetal heart pattern other than initial fetal deceleration. Choice D is incorrect. This strip does not have a normal variability pattern. This strip indicates a non-reassuring finding. Late decelerations become more concerning when they are recurrent (present with 50% or more of the uterine contractions in a 20-minute period). Late decelerations are caused by placental insufficiency. Interventions for late decelerations include - • Maternal repositioning (either lateral or hands and knees) • Intravenous fluid boluses • Administering oxygen (via nonrebreather 10 L/min) • Discontinuing oxytocin infusion (if applicable)

The nurse is reviewing dietary teaching with a client who has hypercalcemia. Which foods should the nurse recommend that the client avoid?

Broccoli 2% milk Seafood Choices A and B. Hypercalcemia can occur in various conditions such as primary hyperparathyroidism, malignancies, milk-alkali syndrome, medications, vitamin D toxicity, and sarcoidosis. Symptomatic hypercalcemia can lead to constipation, psychosis, polyuria, and dehydration. Clients with hypercalcemia should take some dietary precautions to reduce calcium intake. Broccoli is rich in calcium and should therefore be avoided in clients with hypercalcemia. Milk is rich in calcium and should therefore be avoided in clients with hypercalcemia. Choice E. Vitamin D is one substance that, along with parathyroid hormones, regulates a person's calcium levels. Several kinds of seafood are rich in Vitamin D and should be avoided if hypercalcemia is a concern. Choices C and D are incorrect. Whole wheat pasta is not a calcium-rich food. Bananas are particularly high in potassium, not calcium. The nurse does not need to instruct the client with hypercalcemia to avoid whole wheat pasta or bananas.

The nurse is educating clients that are attending a prenatal class. Which of the following statements should the nurse include?

C. "Amniocentesis may be used to assess for chromosomal abnormalities." Choice C is correct. Amniocentesis is a widely used antepartum test that may determine the gender of a fetus, the presence of neural tube defects, chromosomal abnormalities, and fetal lung maturity. This test may also be used therapeutically for polyhydramnios as it may remove some excessive amniotic fluid volumes. Choices A, B, and D are incorrect. CVS is a test that may be performed as early as week ten to determine if the fetus has any chromosomal abnormalities - not neural tube defects. MSAFP is a test that assesses neural tube defects, not gender. A BPP considers five variables, and fetal glucose is not one of these variables. Amniocentesis is utilized to detect fetal neural tube defects and chromosomal abnormalities. This test can also be used later in pregnancy to determine fetal lung maturity. The client will need to consent before the procedure, and if the client is Rh-negative, she will need to receive prescribed RhoGAM. The client should be instructed to report post-procedure fever, decreased fetal movement, leaking of amniotic fluid, or bleeding.

The nurse is providing education to a student nurse assigned to care for a pregnant mother with preeclampsia. The nursing student would not need to be corrected if they said which of the following about this condition?

C. "High blood pressure is one of the findings in preeclampsia and occurs after 20 weeks gestation." Choice C is correct. Preeclampsia occurs when a woman develops high blood pressure after 20 weeks of gestation. Choice A is incorrect. High blood pressure that presents before 20 weeks, usually is preexisting hypertension unrelated to pregnancy. Choice B is incorrect. Kidney function can be affected by high blood pressure but is not the defining factor of preeclampsia. Choice D is incorrect. Frequent nausea and vomiting, which sometimes limit weight gain during pregnancy, are known as hyperemesis gravidarum, not preeclampsia.

The nurse is assessing a client who is newly diagnosed with irritable bowel syndrome (IBS). Which of the following findings is consistent with this diagnosis?

C. Alternating constipation and diarrhea Choice C is correct. Alternating constipation and diarrhea are the hallmark manifestations associated with irritable bowel syndrome (IBS). Choices A, B, and D are incorrect. Unexplained weight loss is a finding associated with multiple diseases, including colon cancer. This is not a finding relevant to IBS. Epigastric pain and nausea may be a symptom associated with pancreatitis. Finally, IBS is not an infectious process, and a fever is not an accurate clinical finding. IBS is a disorder that manifests with alternating periods of constipation and diarrhea. While some clients may have one symptom over another, the disorder is associated with pain with defection (or after defecation), excessive flatulence, and abdominal bloating. The symptoms may relapse and remit and can be triggered by stress or food. Treatment is symptomatic with an emphasis on preventing the occurrence of triggers.

The nurse is caring for a patient who is recovering from open-heart surgery. For the first 24 hours following the surgery, there is a noticeable pinkish fluid oozing from the incision site. Which phase of the inflammatory response does this represent?

C. Exudate formation Choice C is correct. The fluid and white blood cells that leak from blood vessels in response to an injury/inflammation are exudates. Exudates are present in the wounds as they heal. The nature and quantity of exudates depend on the severity of the damage and the tissues involved. For example, a surgical incision may ooze clear or pinkish (serous or serosanguinous) exudate for a day or two. If an exudate becomes purulent (thick, tan, green, or yellow), it is not normal and may suggest infection. In such cases, the nurse should immediately notify the health care provider. Choice A is incorrect. The vascular response of the inflammatory process involves constriction of blood vessels at the injury site immediately after the injury to control bleeding, followed by dilation, increased blood flow to the area (hyperemia), and swelling (edema). Choice B is incorrect. The cellular response of the inflammatory process involves specialized white blood cells (phagocytes) migrating to the site of injury and engulfing bacteria, other foreign material, damaged cells, to ultimately destroy them. The cellular response must occur before the formation of exudate. Choice D is incorrect. Healing is the replacement of tissue by regeneration or repair. Recovery is the replacement of the damaged cells with identical or similar cells. Most injuries heal by repair, wherein scar tissue replaces the original tissue. This is the last phase of the inflammatory response.

The nurse is preparing to infuse prescribed cisplatin to a client with cancer. Which priority assessment should the nurse make before administration?

C. Intravenous (IV) patency Choice C is correct. IV patency is critical to assess before the infusion of a chemotherapeutic. Serious injuries (extravasation) have been caused by medications like cisplatin being infused into a nonpatent vascular access device. Central lines are highly recommended when infusing chemotherapeutic drugs like cisplatin. Choices A, B, and D are incorrect. Cancer staging is not a critical assessment before the infusion of cisplatin. This would not impact the actual administration of the drug. While blood dyscrasias (low hemoglobin and hematocrit) may occur with cisplatin, this would not be an essential assessment. Finally, electrolyte disturbances may occur with cisplatin, but it would not be critical before starting the infusion.

The nurse is planning a staff development conference about medication reconciliation. Which of the following information should the nurse include?

C. This process should occur at admission, client transfer, and discharge. ✓ Medication reconciliation is an essential process designed to promote client safety. ✓The client's complete list of medications (including over-the-counter medications) should be collected during this process. ✓This process should occur at admission, client transfer, and discharge. ✓ The nurse must obtain the most recent medications from the client and their adherence (for example, a client is prescribed omeprazole, but the client indicates that they do not take the medication). ✓This process should be thorough and involve the client.

The nurse is caring for a client immediately postoperative following a below-the-knee amputation. The nurse should take which priority action?

Check the operative site for bleeding Choice B is correct. A complication following an amputation is hemorrhage. An early assessment and action of the nurse is to monitor the client for hemorrhage, which may be evident on the bandage, or if the client has a drain, a large amount of bloody drainage may be apparent. Findings that may support that the client is hemorrhaging include tachycardia with later development of hypotension. Rather the amputation is traumatic or surgically performed; it is hemorrhage that is a concerning complication. Choices A, C, and D are incorrect. These are all appropriate actions the nurse should take regarding caring for a client with a below-the-knee amputation. They do not prioritize the client's potential hemorrhage, which is a significant concern postoperatively. A client's mobility will be limited, and physical therapy should be consulted to start exercises, including bed mobility. A trapeze should be added to the bed to promote mobility and condition the upper extremities. Depending on the physician's order, most below-the-knee amputations have the stump elevated for the first 24 hours to decrease postoperative edema. After this, the client may be prone for 20-30 minutes four times daily to prevent a flexion contracture. The client should be taught how to use the IS because this helps decrease hypostatic pneumonia, a risk because of the client's limited mobility. ✓ Following an amputation, hemorrhage is an early complication that must be surveilled by looking at the dressing, drains, and trending the vital signs (tachycardia would be an early manifestation) ✓ Other issues surrounding an amputation include phantom limb pain, contractures, and depression

The nurse is preparing a patient for scheduled total knee arthroplasty (TKA). Which action by the nurse would be most important to reduce this patient's risk for experiencing emergence excitement after this procedure?

Choice A is correct. Patients who are anxious prior to anesthesia are at higher risk of experiencing postoperative emergence excitement or delirium. The nurse should focus on actions that aim to reduce the patient's anxiety to reduce this patient's risk of emergence excitement. The nurse should provide reassurance, explain the purpose of procedure, and allow the patient to express concerns/ask questions.

The nurse is admitting a client for treatment of C. difficile. Which sign should the nurse affix to the client's door?

Choice C is correct. Contact special enteric is the appropriate precaution for Clostridium difficile. Clostridium spores are transmitted by direct contact but are not killed with an alcohol-based hand sanitizer. Handwashing with soap and water is required after exiting a room under special enteric precautions. Choice A is incorrect. Airborne precautions are not appropriate for Clostridium difficile infection. This type of care is used for diseases transmitted by microbes that can survive in the air and can spread beyond three feet from the client, such as tuberculosis, chickenpox, disseminated herpes zoster, and severe acute respiratory syndrome (SARS), and measles (\rubeola). Choice B is incorrect. Regular contact precautions signage does not specify handwashing requirements with soap and water. Regular contact precautions allow hand sanitization with either alcohol-based hand sanitizer or soap and water. Hence, this signage with no specific handwashing requirement is not appropriate for Clostridium difficile infection. This type of signage is used for diseases transmitted by direct contact with the client or their secretions, not those that involve spores. Infections such as MRSA, shigella, salmonella, scabies, localized herpes zoster, hepatitis A, respiratory syncytial virus (RSV), pediculosis, and vancomycin-resistant enterococci (VRE) are appropriate for the usual contact precautions signage. Choice D is incorrect. Droplet precautions are not appropriate for Clostridium difficile infection. This type of transmission precaution is used for diseases that are transmitted by droplets up to three feet away from the client, such as influenza, mumps, rubella, adenovirus, pertussis, and bacterial meningitis. LEARNING OBJECTIVE Understand that handwashing is a necessary requirement in controlling infections transmitted by the spore-forming bacteria ( Clostridium difficle). ADDITIONAL INFO According to the Centers for Disease Control, the transmission of C. diff can be disrupted through: Meticulous hand hygiene with soap and water. Avoid using alcohol-based hand sanitizers. Using disposable healthcare equipment, such as blood pressure cuffs and stethoscopes. Disinfect surfaces with a bleach solution. Discontinuing unnecessary antibiotics.

Which of the following over-the-counter (OTC) medications is Reye's syndrome associated with?

Choice C is correct. Reye's syndrome is a potentially fatal illness that can lead to liver failure and encephalopathy. Virus-infected children who are given aspirin to manage pain, fever, and inflammation are at an increased risk of developing Reye's syndrome. Choice A is incorrect. The use of acetaminophen has not been associated with Reye's syndrome and can be safely given to patients with fever due to viral illnesses. Choice B is incorrect. Ibuprofen's adverse effects include GI irritation and bleeding; in toxic doses, both renal and hepatic failure are reported. However, ibuprofen has not been associated with the onset of Reye's disease. Choice D is incorrect. Brompheniramine/pseudoephedrine contains a first-generation OTC antihistamine and a decongestant. Neither agent has been associated with the development of Reye's syndrome.

The nurse is providing asthma education to a teen that has just been diagnosed with asthma. Which of the following statements by the client would indicate a need for further teaching? Select all that apply.

Choices A, D, E, and F are correct. A is correct. This statement indicates a need for further education. During an asthma attack, the first action should not be to call 911. The client will have an asthma action plan listing the steps that should be taken in the correct order. For most clients, the first step is to take short-acting inhaler medications. It is not necessary to first call 911 for every asthma attack as symptoms often improve with less aggressive interventions. D is correct. This statement indicates a need for further education. Although asthma will not stop every child from getting a dog, pets with hair that sheds can be a trigger. It would be inadvisable for a teen newly diagnosed with asthma to get a new dog, as this could end up causing more asthma attacks and present a severe problem. If the client wants a new pet, a fish would be a better recommendation given their new asthma diagnosis. E is correct. This statement indicates a need for further education. When administering albuterol via an inhaler, the client should not take a quick or shallow breath. Instead, when delivering the dose, the client should be taught to inhale slowly and deeply to help the medication reach the small airways in the lungs. F is correct. This statement indicates a need for further education. Theophylline is a bronchodilator that works as a long-term medication for asthma. This medication must be taken regularly to be effective and does not work immediately so would not be useful if only taken prior to exercising. Choice B is incorrect. This is an appropriate statement and does not indicate a need for further education. When a client is having an asthma attack, the physiology includes inflammation and constriction of the airways. This can result in obstruction, making it impossible for the client to breathe. That is why asthma attacks are so dangerous. Choice C is incorrect. This is an appropriate statement and does not indicate a need for further education. One of the most critical educational points for clients newly diagnosed with asthma is identifying their triggers. Triggers are what precipitate an asthma attack for that patient (eg. playing soccer, dusting the house). Whatever it is that precipitates their asthma should be avoided.

The nurse encounters an infant with irritability from acute otitis media while working in the pediatric clinic. The nurse should know that the infant is at much higher risk than an adult for otitis media due to which of the following?

D. Narrower, shorter, and more horizontal Eustachian tubes Choice D is correct. Infants have more horizontal, shorter, and narrower eustachian tubes, which makes them more prone to otitis media. The eustachian machine is a conduit from the middle ear to the nasopharynx. An inflammatory swelling in the eustachian tube can cause it to be blocked, trapping the fluid in the middle ear and eventually leading to infection. Several factors, such as allergies, common cold, viral flu, sinus infection, enlarged adenoids, and drinking while lying down (in infants), may predispose to swelling/ blockage of the eustachian tube. In an adult, the eustachian tube typically measures 36 mm and is angled at 45 degrees. In infants, it is shorter (18 mm) and has a more horizontal (angle at 10 degrees). Such a shorter tube predisposes to infection via reflux of bacteria from the nasopharynx Choice A is incorrect. An immature cardiac sphincter causes vomiting, not otitis media. Choice B is incorrect. Feeding in a semi-Fowler position helps decrease the risk of otitis media because the infant is not lying flat. Choice C is incorrect. The introduction of solid foods has no bearing on the incidence of otitis media.

During a client's bronchoscopy, the nurse notices the client necessitates the use of a rigid scope. During the insertion of the scope by the health care provider (HCP), the nurse notes the client is experiencing a vasovagal response when noticing which of the following?

D. Noticeable drop in heart rate Choice D is correct. The nurse would notate a vasovagal response upon seeing a sudden, noticeable drop in the client's heart rate. Here, during the bronchoscopy, the involvement of a foreign object (i.e., the HCP's scope) in the client's pharynx likely caused vagus nerve stimulation. This stimulation resulted in a vasovagal response by the client, manifested by a sudden decrease in the client's heart rate. Choice A is incorrect. Stimulation of the vagus nerve does not cause dilation of the pupils. Choice B is incorrect. Stimulation of the vagus nerve does not cause bronchodilation. Choice C is incorrect. During vagus nerve stimulation, an increase, not decrease, in gastric secretions is typically noted. This response occurs as a result of increased parasympathetic activity. Increased vagal firing (i.e., increased parasympathetic activity) at the sinus node and the atrioventricular node causes a decrease in heart rate. This decrease in heart rate can be profound, with asystole lasting as long as several seconds. Vagal stimulation may result in bradycardia, hypotension, heart block, ventricular tachycardia, or other dysrhythmias.

The perinatal nurse is caring for a client experiencing suspected placental abruption. Which of the following signs and symptoms would be expected assessment findings for this client?

Dark red bleeding Hypotension Rigid abdomen Choices B, C, and D are correct. B is correct. A massive amount of dark red bleeding is a prominent sign of placental abruption. The bleeding occurs due to the placenta separating from the wall of the uterus. C is correct. Due to the massive amounts of dark red bleeding, hypotension is a sign of placental abruption. When the mother loses large amounts of blood, her blood pressure will drop, potentially resulting in hypovolemic shock. This hypovolemia is treated with IV fluids and blood products such as PRBCs. D is correct. A rigid, board-like abdomen is a sign of placental abruption. This is also due to massive blood loss and internal bleeding. As the placenta separates from the womb's wall, blood accumulates in the abdomen, causing it to become rigid and board-like. Choice A is incorrect. Painless bleeding is not a sign of placental abruption. Instead, it is a sign of placenta previa. In placenta previa, the placenta covers the cervix, causing painless bleeding. In placental abruption, the placenta separates from the wall of the womb, causing a massive amount of very painful, dark red bleeding. Choice E is incorrect. Fetal tachycardia would not be an expected assessment finding with placental abruption. Instead, fetal bradycardia and decelerations would be expected due to fetal distress as perfusion to the fetus decreases dramatically. Maternal tachycardia may be present due to hypovolemia from blood loss. ✔︎ Placental abruption (also called abruptio placentae) describes the separation of the placenta prior to delivery. ✔︎ Placental abruption can present as either incomplete (partial detachment of placental) or complete (full detachment of placental). Incomplete placental abruption causes internal bleeding due to pooling of blood behind the placenta. Complete placental abruption is extremely painful and causes significant external bleeding. ✔︎ Placental abruption poses a risk to both the mother (hemorrhage, hypovolemic shock, clotting issues) and the fetus (asphyxia, blood loss, prematurity).

The nurse is caring for a client with end-stage renal disease who receives prescribed sevelamer. Which of the following findings would indicate a therapeutic response?

Decreased serum phosphorus levels Choice D is correct. Sevelamer is a phosphate binder indicated in the treatment of hyperphosphatemia associated with chronic kidney disease. This medication is purported to decrease serum phosphorus levels by binding to food. Thus, this medication is given with meals. Combined with a low phosphorus diet, the goal of this medication is to decrease serum phosphate levels. Choices A, B, and C are incorrect. The goal of this medication is to decrease serum phosphorus, thereby raising serum calcium (calcium and phosphorus have a reciprocal relationship). Hyperphosphatemia is a common problem associated with chronic kidney disease and end-stage renal disease, and medications such as sevelamer are used in its management. These medications are not indicated for raising hemoglobin or decreasing serum potassium. Hyperphosphatemia is a common problem associated with chronic kidney disease and end-stage renal disease, and medications such as sevelamer are used in its management. ➢ Management is with restricting dietary phosphorus combined with oral phosphate binders. ➢ Food sources with high levels of phosphorus include beans, fish, and nuts. ➢ These phosphate binders may be calcium-containing (calcium carbonate) and noncalcium-containing (sevelamer). ➢ These medications are only effective when taken with meals. ➢ Major side effects of these medications include constipation which may lead to paralytic ileus. ➢ Other side effects include vitamin deficiencies which is why a renal vitamin may be prescribed. ➢ Calcium and phosphorus levels should be monitored closely during the treatment.

Which of the following are management functions that nurse managers fulfill?

Directing Planning Organizing Staffing Choices B, C, D and E are correct. The five major management functions are planning, organizing, staffing, directing, and controlling. Each of these functions are discussed in detail in the additional information section below. Choice A is incorrect. Although effective leadership often utilizes empowerment, it is important to note that leadership and management differ from one another. Empowering others is not considered a management function. As noted above, the five major management functions are planning, organizing, staffing, directing, and controlling. Of note, some sources list coordinating in place of staffing. Planning describes the decision-making process regarding what needs to be done, how it will be done, and what resources/individuals will be utilized in the process. This process is continuous, with the nurse manager needing to continuously reassess the process and evaluate if changes are needed. Organizing refers to the organizational structure which determines the chain of command and the method decisions are made. Staffing is the acquisition and management of adequate staff as well as the correct staffing combination. Directing refers to a leadership role assumed by a manager which influences and motivates the staff to perform assigned roles or duties. Although empowerment may be one method a manager may utilize when directing, leaders are more likely to use empowerment than managers. Controlling involves continuously evaluating data (i.e., staff performance, feedback, and other measurable data) to ensure previously identified outcomes are being met and, if needed, making necessary adjustments

The nurse is caring for a client who has rubella. The nurse should isolate the client using which of the following?

Droplet precautions Choice B is correct. Rubella is known as German measles and requires droplet precautions. The nurse is right to wear a surgical mask when engaging with the client. The transmission mode for rubella is a droplet mode of communication where the spread occurs with particle drops larger than 5 microns. Choices A, C, and D are incorrect. Do not confuse rubeola with rubella. Rubeola (measles) requires airborne precautions, whereas rubella needs droplet precautions. Airborne transmission occurs when the pathogen is carried in dust or droplets in the air that remains in place for sufficient time to infect a person exposed to this air. Contact transmission precautions are used for infectious diseases spread by contact with the client or the client's environment. Contact transmission precautions prevent infections such as C. difficile and shigella, but not rubella. Standard precautions are a set of infection control practices used to prevent the transmission of diseases that can be acquired by contact with blood, body fluids, non-intact skin (including rashes), and mucous membranes. Droplet precautions involve wearing a surgical mask and are used for rubella. Standard precautions do not include routinely wearing a surgical mask and are insufficient to prevent rubella transmission.

The nurse is speaking with a client that underwent an ileostomy one month ago. The client states that the ostomy's odor is causing embarrassment. The nurse investigates the food that the client consumes and initiates further health teaching when the client mentions the following menu item:

Eggs Choice D is correct. Eggs, asparagus, alcohol, fish, onions, cabbage, and grapes are just a small sampling of the foods known to cause foul-smelling stools, gas, and/or increased foul odors, which ostomy clients often report as concerning. These clients should be educated and provided with a comprehensive list of foods to avoid to reduce the likelihood of omitting foul odors from their ileostomy. Choice A is incorrect. Buttermilk helps alleviate gas and odor in ileostomy clients. Therefore, it is considered a deodorizing food for these clients. Choice B is incorrect. In ileostomy clients, parsley is considered a deodorizing food, as it helps alleviate gas and odor. Choice C is incorrect. Yogurt (with active cultures) is considered a deodorizing food, as it often helps relieve gas and odor in clients with an ileostomy.

The nurse performs a physical assessment on a client and observes the following finding while the client has their arms extended. The nurse understands that this finding is consistent with which of the following?

End-stage renal disease Choice B is correct. Asterixis is a hand flapping tremor that may be elicited by having the client close their eyes, extend their arms, dorsiflex their wrist, and spread their fingers. End-stage renal disease causes azotemia and may trigger this unilateral or bilateral tremor in end-stage renal disease. While this tremor is poorly understood, it is likely the accumulation of nitrogenous waste that contributes to the development of this action. This tremor has also been associated with moderate to severe hepatic encephalopathy. Choices A, C, and D are incorrect. Neuroleptic Malignant Syndrome is an insidious autonomic reaction that adversely causes muscle rigidity, tachycardia, and pyrexia. NMS is commonly triggered by antipsychotics and causes hyporeflexia; thus, it would not cause asterixis. HIV is an infectious disease that does not produce asterixis. Rheumatic fever is a condition characterized by arthritis, carditis, and chorea. While an individual with rheumatic fever may have motor disturbances, the Sydenham chorea produces a dance-like motion overtly seen during gross motor movements. These brief shock-like movements may be associated with conditions such as hepatic encephalopathy, end-stage renal disease, and drug intoxication with phenytoin. Most asterixis is bilateral but unilateral asterixis may develop because of pathology in the brain.

The physician orders the client to go to surgery for a laparoscopic appendectomy. The physician obtains the client's informed consent from her parents. The nurse provides education regarding this surgery. ➢ Which two (2) client statements indicate that the client needs further post-operative education?

Following a laparoscopic procedure, the client will need a recovery period of up to one week. The client going to school tomorrow is not realistic and requires follow-up. The client's diet will be advanced per the physician and will be advanced over a couple of days - not weeks. Laparoscopic procedures involve the process of insufflation to visualize organs. Gastric distention and cramping are common and mitigated with ambulation. Pain medication will be prescribed after this surgery, and to prevent complications such as pneumonia; the client will need to turn and perform deep breathing exercises.

The nurse is preparing a staff in-service regarding conductive hearing loss. It would be appropriate for the nurse to identify which factors cause this type of hearing loss? Select all that apply.

Foreign body Cerumen Choices C and E are correct. Conductive hearing loss is typically reversible and caused by cerumen, foreign body, tumor, edema, and acute infection. Hearing loss is classified into three types: conductive, sensorineural, and mixed. Conductive hearing loss is caused by an obstruction to the sound transmission within the ear. Causes of this type of hearing loss include cerumen, foreign body, water, edema, infection (otitis media), or tumor. This type of hearing loss may be reversible. Sensorineural hearing loss is caused by damaged hair cells within the inner ear or the impairment of the vestibulocochlear nerve. Causes of this type of hearing loss include prolonged exposure to noise, ototoxic substances (aminoglycosides), diabetes mellitus, and presbycusis (age-related hearing loss). This type of hearing loss is often not reversible. Mixed hearing loss has both conductive and sensorineural components. Causes include head trauma, genetic disorders, and infections.

Due to a recent flood, the only staff that were able to make it to work are two nursing assistants and one licensed practical nurse with the nurse manager. Knowing the different nursing delivery systems, which system should the nurse manager implement to care for the 20 clients admitted in their ward?

Functional nursing Choice C is correct. In functional nursing, each caregiver on a specific nursing unit is given specific tasks that fall into their scope of practice. In this situation, the nurse manager may administer medications to the entire group, while a licensed practical nurse performs treatments, and the client care attendants provide physical care. Choice A is incorrect. A registered nurse plans and organizes care for a group of clients and cares for this group during their entire hospitalization. This type of care delivery cannot be useful in this situation. Choice B is incorrect. An RN leads nursing staff who work together to provide care for a specific number of clients. The team typically consists of RNs, LPNs, and client care attendants. The team leader assesses client needs, plans client care, and revises the care plan based on changes in the client's condition. The leader assigns tasks to team members as needed. This cannot be done in this situation, as this requires too many staff members. Choice D is incorrect. Case management is a form of primary nursing that involves a registered nurse who manages the care of an assigned group of clients. This nurse coordinates care with the entire health care team. There is only one RN in the situation. Therefore, it cannot be used.

The nurse is caring for a client with peritoneal dialysis. The client reports an outflow of only one-half of the dialysate solution that was dwelled. The nurse should instruct the client to do which of the following?

Have a bowel movement. Choice B is correct. Constipation is a significant problem associated with peritoneal dialysis. Constipation comes from the consumption of prescribed phosphate binders as well as chronic kidney disease itself. The client having a bowel movement allows decreased intestinal pressure, therefore resolving the outflow failure. If constipation is a significant problem causing outflow failure, the client can give themselves an enema before the procedure. Peritoneal dialysis offers many advantages, such as the ability for the client to remain at home for the procedure, fewer dietary restrictions, and fewer hemodynamic complications. The number one complication associated with peritoneal dialysis is peritonitis. This complication may be mitigated through an aseptic technique when handling the catheter.

The nurse is assessing a child with lead poisoning. The nurse should expect which findings?

Headaches Irritability Abdominal pain Peripheral neuropathy Choices B, C, D, and E are correct. All of these answer choices are possible symptoms related to lead poisoning. Even small amounts of lead can cause serious health problems Signs and symptoms associated with lead poisoning include learning problems, brain damage, peripheral neuropathy, hyperactivity, headache, and hearing loss. Other symptoms and signs can consist of slow growth, irritability, fatigue, baby colic, memory loss, constipation, vomiting, or nausea. Choice A is incorrect. Lead is a heavy metal, a neurotoxin, and is not cardiotoxic. Children younger than six years are especially vulnerable to lead poisoning, which can severely affect mental and physical development. This is because the blood-brain barrier has not been fully developed. Lead is a neurotoxin and may cause hearing loss, peripheral neuropathy, acute encephalopathy, and cognitive dysfunction at high levels. At very high levels, lead poisoning can be fatal.

The nurse is educating nursing students about factors that can influence cardiac output. Which of the following would cause an increase in cardiac output?

Increased stroke volume Increased blood volume Increased sympathetic stimulation Administration of positive inotropic drugs A is correct. Stroke volume refers to the amount of blood ejected by the left ventricle during each contraction. An increase in stroke volume would directly increase cardiac output, as it is one of the two factors determining cardiac output (Workman, 2021). B is correct. An increase in blood volume can lead to an increase in cardiac output. More blood volume can stretch the heart muscle fibers, leading to a more muscular contraction and stroke volume, thereby increasing cardiac output (Frank-Starling law) (Workman, 2021). C is correct. Sympathetic stimulation increases both heart rate and the force of myocardial contraction, which can increase cardiac output. This is part of the body's 'fight or flight' response (Workman, 2021). D is correct. Positive inotropic drugs, such as digoxin, increase the force of myocardial contraction. This can lead to increased stroke volume and cardiac output (Workman, 2021). Choice E is incorrect. E is incorrect. Increased systemic vascular resistance, or afterload, actually decreases cardiac output. Increased SVR means the heart must work harder to pump blood into the systemic circulation, which may decrease stroke volume and reduce cardiac output.

The nurse reviews laboratory work ordered by the primary healthcare provider (PHCP) The nurse obtains physician orders for this client with HHS ➢ The nurse is preparing to implement the physician's orders. Which order should the nurse clarify with the physician?

Infuse two liters of 0.9% saline over one hour The priority treatment for a client with HHS is the repletion of fluids as the client is grossly dehydrated. The clinical standard is one liter of isotonic saline per hour and assessing the client's electrolytes throughout the correction. The client has a history of congestive heart failure, and two liters of saline in one hour may be highly detrimental, and this order should be clarified with the PHCP. Finally, the nurse should recognize that correcting the dehydration should be done slowly because of the client's sodium level. A correction that goes unmonitored or too fast may cause cerebral edema. The client's potassium level is low, and the order for potassium replacement is appropriate. Potassium depletion is common in HHS because of the metabolic alkalotic state it causes. Obtaining capillary blood glucose every two hours for a client with HHS is appropriate. Hourly glucose monitoring should be performed for a client receiving a continuous insulin infusion.

The nurse is performing a verbal hand-off report for a client. Which essential information should the nurse include in the report?

Involuntary admission status Choice B is correct. Admission status is essential information provided in the hand-off report because involuntary admission requires the client to stay in the healthcare facility. This status is typically required when a client may pose a threat to themselves or others. This type of involuntary admission status also may raise the risk of the patient eloping. This should be communicated because if a client is involuntarily admitted, they may not have a rational thought process which may raise the risk of self-injury if they do successfully elope. Choices A, C, and D are incorrect. The current medication list is generally not communicated during the hand-off report. Hand-off reports should include new prescriptions or prescriptions pertinent to the client's care. The oncoming nurse may easily obtain this list by accessing the medication administration record. Food and mealtime preferences are important to delivering client-centered care but do not prioritize the client's admission status. Finally, the presence of family at the bedside may be irrelevant unless pertinent family dynamics impact care.

For each client finding below, click to specify if it is consistent with cholecystitis, pregnancy, or appendicitis. Each finding may support more than one condition. Each column must have at least one response option selected.

Many overlapping features exist between appendicitis and cholecystitis. Both cause nausea & vomiting, fever, and anorexia. The pain differs where appendicitis causes more periumbilical pain, and cholecystitis causes pain in the right upper quadrant that radiates to the upper scapula. Pregnancy only shares the early manifestation of nausea and vomiting. Abdominal cramping may be associated with early pregnancy signs, but not abdominal pain localized to the periumbilical area.

A man is found lying on the ground, confused, covered with snow, and is brought to the emergency department to be treated. The nurse checks the client's temperature and notes that it is 88° F (31.1 °C). The nurse also notes respirations of 10, a pulse of 50, and blood pressure of 79/52 mm Hg. The nurse correctly identifies that the client is experiencing which of the following conditions?

Moderate hypothermia Choice B is correct. Hypothermia is defined as a core body temperature of less than 95° F ( 35°C). Hypothermia is staged into Mild, moderate, severe, and profound hypothermia ( stages I to IV). Staging helps guide the treatment recommendations. A core body temperature between 90 to 95° F (32.2°C to 35°C) is considered mild hypothermia whereas a temperature between 82° F to 90° F (27.8°C to 32.2°C) is considered moderate. A core temperature less than 82° F (27.8°C) is severe. However, measuring the core body temperature accurately is challenging. Therefore, a model based on the vital signs and clinical symptoms called the "swiss staging model" is used to stage hypothermic patients. Based on the clinical manifestations as well as the stated core body temperature of 88° F given in this question, this client has moderate hypothermia ( Stage II). Manifestations of moderate hypothermia include decreased level of consciousness (LOC), hypoventilation, bradycardia, atrial fibrillation, hypovolemia, cessation of shivering, and possible hyperglycemia. Choice A is incorrect. Mild hypothermia (Stage I) presents with shivering, bradycardia, or tachycardia. The patient may also be alert or slightly confused. Choice C is incorrect. Severe hypothermia (Stage III) manifests as coma, fixed and dilated pupils, bradycardia, apnea, hypotension, ventricular fibrillation, and/or asystole. The core body temperature is typically less than 82° F Choice D is incorrect. Frostbite is hypothermia in the extremities. Frostbitten areas may appear red and swollen or may be pale in color. Blisters containing clear or bloody dark fluid may appear.

The nurse is caring for a client who is receiving prescribed fentanyl. Which of the following findings would indicate the client is having a side effect? Select all that apply.

Nausea and vomiting Constipation Pruritus Urinary retention Fentanyl is an opioid analgesic used to manage acute and chronic pain. Common effects associated with this drug include nausea and vomiting, constipation, pruritus, and urinary retention. Choice E is incorrect. Nystagmus is not associated with fentanyl. Ophthalmic effects associated with fentanyl include blurred vision and miosis.Fentanyl is an opioid analgesic used to manage acute and chronic pain. Common effects associated with this drug include nausea and vomiting, constipation, pruritus, and urinary retention. Choice E is incorrect. Nystagmus is not associated with fentanyl. Ophthalmic effects associated with fentanyl include blurred vision and miosis.

The nurse cares for a client at 30 weeks gestation at risk of delivering preterm. Which of the following medication would the nurse anticipate the primary healthcare provider (PHCP) to prescribe?

Nifedipine Choice B is correct. Nifedipine is a calcium channel blocker indicated as a tocolytic in preterm labor. This medication relaxes smooth muscle and reduces uterine contractions. Choices A, C, and D are incorrect. Penicillin G is indicated in treating maternal infections such as syphilis or infective endocarditis. Oxytocin would be contraindicated for preterm labor as it causes uterine contractions. Misoprostol is another medication that causes uterine contractions and is frequently utilized in miscarriages. This medication would not inhibit preterm labor.

The nurse is positioning a client following a liver biopsy. Which position is best suited for this client?

On the right side with a pillow under the biopsy site. Choice D is correct. The client should lay on the right side with a pillow against the biopsy site at the right costal margin. Following the liver biopsy, the right lateral decubitus position reduces the risk of post-biopsy bleeding by putting pressure on the liver biopsy site. Choice A is incorrect. Clients who have just had a liver biopsy should maintain pressure on the biopsy site. Laying laterally on the left side with a pillow underneath the ribs does not provide enough support to the actual location of the liver biopsy because the liver is located on the right side. Choice B is incorrect. The supine with a pillow under the client's hips is not the best. This position does not provide any support to the site of the liver biopsy. Choice C is incorrect. Facedown, or prone, is not an appropriate way to lay after a liver biopsy.

The nurse is assessing a client for bacterial meningitis. Which of the following assessments should the nurse perform?

Oral temperature Glasgow Coma Scale Choices A and D are correct. Bacterial meningitis manifests as a stiff neck, photophobia, fever, altered mental status, and malaise. The nurse would need to perform an oral temperature and the Glasgow Coma Scale to discern the client's current mental status. Choices B, C, and E are incorrect. Patellar reflexes are not significant in the assessment of a client with bacterial meningitis. Patellar reflexes would be pertinent for a client with a severe electrolyte imbalance such as hypermagnesemia. Weber and Rinne testing are utilized to differentiate between the types of hearing loss and is not pertinent to meningitis. A client with meningitis may have low blood pressure due to the dehydration caused by the fever. However, an orthostatic assessment of their blood pressure is not warranted.

Client Findings(Appendicitis,Pancreatitis,Cholecystitis) Nausea and Vomiting Epigastric tenderness Signs of dehydration Pain triggered by alcohol intake

Overlapping features of appendicitis, pancreatitis, and cholecystitis include abdominal pain, dehydration, and nausea/vomiting. A gallstone or alcoholism may trigger pancreatitis. Additionally, pancreatitis may cause a client to have epigastric or left upper quadrant tenderness. Because all three conditions feature nausea and vomiting, dehydration is highly likely in all conditions.

Complete the diagram by dragging from the choices below to specify what condition the client is most likely experiencing, two (2) actions the nurse should take to address that condition, and two (2) parameters the nurse should monitor to assess the client's progress.

Potential Conditions This client is experiencing significant hypothyroidism and requires immediate treatment. The clinical features that the client is demonstrating supporting hypothyroidism included periorbital edema, fatigue, bradycardia, hypotension, weight gain, constipation, anemia, increased TSH and decreased T3/T4. While adrenal insufficiency may cause hypotension and fatigue, it would not alter the thyroid hormones. Graves' disease is the most common form of hyperthyroidism and manifests as tachycardia, weight loss, and heat intolerance. Lupus is an inflammatory condition marked by fatigue, muscle and joint pain, anemia, and changes to the integument. The client is not reporting any pain, and while she does have anemia, it is related to thyroid abnormalities. Cushing's syndrome would be excluded because while this does cause weight gain and edema, it would not be explained by the altered thyroid levels. Action to Take This client's hypothyroidism is significant and requires the nurse to obtain a prescription for levothyroxine. Additionally, the client has low blood pressure and endorses dizziness; this should prompt the nurse to keep the client safe and implement fall precautions. Obtaining urine cortisol levels would be done for issues with the adrenal (Addison's / Cushing's) and not for this client. Additionally, this client does have anemia, but a transfusion of packed red blood cells is indicated when the hemoglobin is 7 g/dL or less. Anemia is expected with hypothyroidism. Methimazole would be helpful for Graves' disease, which this client is not experiencing. Parameters to Monitor The client arrives both with bradycardia and low blood pressure. Monitoring the vital signs is essential. Further, the nurse must monitor the serum thyroid levels. This would prevent overmedication which would cause the client to develop hyperthyroidism. While the client's sodium is marginally low, this is not a priority. The client's BUN and creatinine were within normal limits and did not require monitoring. This is not an adrenal problem, so cortisol would not be a parameter that needs to be monitored. Intake and output would not be essential parameters to monitor, as weight would be the greatest indicator of th

The nurse is evaluating the lab test results of one of her prenatal clients. She is eight weeks along and has a hematocrit level of 36% (Male: 42-52% / Female: 37-47%) and hemoglobin of 11.7 gm/dL (Male: 14-18 g/dL / Female: 12-16 g/dL). These numbers are down from her pre-pregnancy H and H levels. The priority action of the nurse would be to:

Record these normal findings and confirm that the client is on a prenatal vitamin during her next visit. Choice B is correct. These results are typical and should be recorded as such. A drop from pre-pregnancy values is an expected phenomenon if they remain within or close to the normal range. Most women see a decrease in their hemoglobin and hematocrit levels during pregnancy. This phenomenon is known as physiological anemia and occurs as a result of increased plasma volume in the maternal bloodstream. It is essential to confirm that the client is taking prenatal vitamins. Demand for iron increases during pregnancy. Folic acid supplementation is necessary to prevent fetal neural tube defects. Prenatal vitamins will serve to address those needs. Normal hemoglobin in a pregnant client is > 11 g/dL. Normal hematocrit in a pregnant client is > 33%.

A woman is in the labor and delivery suite at 37 weeks gestation. She has been under her obstetrician's care for preeclampsia. The labor nurse notices that the fetus is experiencing heart rate decelerations. You are part of the neonatal resuscitation team that responds to the call from the labor room nurse. The infant is born but does not respond to tactile stimulation. The group moves the infant to the warmer. You evaluate the infant and confirm he is still not breathing. You begin positive pressure blowing with room air. Another team member notes that the heart rate is 72 bpm and the newborn's chest is not moving with PPV on room air. The next appropriate action is to:

Reposition the infant to open the airway Choice A is correct. Reposition the infant to open the airway while ensuring that you have a good seal with the mask on the newborn's face. Following that action, a team member should suction the infant's mouth and nose. Until the team establishes sufficient ventilation, there is no indication to increase oxygen concentration or begin CPR. The AHA and AAP focus on positive-pressure ventilation as the single most crucial step in the resuscitation of the newborn.

The nurse observes a fire in a client's room. The nurse should take which initial action?

Rescue the client Choice A is correct. Client safety is a priority, and the nurse should rescue (or evacuate) the client who is closest to the fire. Choices B, C, and D are incorrect. Sounding the alarm and extinguishing the fire is important after the client is rescued/evacuated. The nurse should not place a linen blanket over the fire unless it is a fire blanket. The linen blanket may make the fire worse. ADDITIONAL INFO ✓ R—Rescue and remove all clients in immediate danger. ✓ A—Activate the alarm. Always do this before attempting to extinguish even a minor fire. ✓ C—Confine the fire by closing doors and windows and turning off oxygen and electrical equipment. ✓ E—Extinguish the fire with an appropriate extinguisher.

A 28-year-old post-thyroidectomy client is transferred from the post-anesthesia care unit to the medical-surgical floor. Which action demonstrates the medical-surgical nurse understands the possible complications of a thyroidectomy?

The bedside is prepared with a tracheostomy set, oxygen, and suction. Choice C is correct. Following a thyroidectomy, one of the most serious complications is an ineffective airway due to tracheal compression, hematoma, and/or edema. Therefore, it is essential to have a tracheostomy set, oxygen, and suction available at the bedside for at least the initial 24-hour post-operative period. Choice A is incorrect. Dressing changes are performed as needed for bleeding. However, post-thyroidectomy bleeding may not be visible on the dressings. Discrete signs, such as cervical pressure and tightness, difficulty swallowing, and subjective shortness of breath are possible signs of bleeding, which may precede a blood-soaked dressing. The majority of post-operative clients are at risk for post-operative bleeding, with many experiencing signs and symptoms of post-operative bleeding prior to the late sign of blood presenting on the bandage. Choice B is incorrect. Pain control in the post-operative period is a concern following all surgical procedures, not simply thyroidectomies. Narcotics are typically administered on a PRN basis once the client has been transferred to the floor. Therefore, it is not appropriate to automatically administer a narcotic medication to the client upon the client's return to the room from the post-anesthesia care unit without first assessing the client and determining the time, dose, and response of the last narcotic medication given to the client. Choice D is incorrect. Although clients may have difficulty communicating due to laryngeal edema or nerve damage, difficulty communicating most commonly occurs due to endotracheal intubation. The client will still be able to speak, but may experience hoarseness of the voice. Post-endotracheal intubation hoarseness of the voice is an issue experienced by all clients who undergo procedures requiring intubation, not just thyroidectomy clients.

The physician orders an abdominal ultrasound and laboratory tests The nurse reviews the results and plans interventions for this client

The client has a high white blood cell count, making it concerning for peritonitis. Her HCG is negative, which excludes pregnancy. For a client with a ruptured appendix, it is essential that the nurse obtain immediate peripheral vascular access, obtain blood cultures, administer broad spectrum antibiotics, and place the client on NPO status because of their likely surgery. A barium enema is not indicated in the treatment or diagnosis of acute appendicitis as it has no relevance.

The client's neurological symptoms may be explained by decreased levels of cobalamin due to reduced production of intrinsic factor

The client is experiencing manifestations associated with Vitamin B12 (cobalamin) deficiency. This is because the client's food bypasses the parietal cells on the stomach lining. The parietal cells secrete intrinsic factor which assists in the absorption of cobalamin. The client's manifestations are consistent with Vitamin B12 deficiency anemia. While the client has a medical history of diabetes mellitus, diabetes does not cause upper and lower extremity paresthesias, forgetfulness, or fatigue. The client did not report any food intolerances beyond feeling fuller sooner, which is expected. Dumping syndrome would be triggered by the client's exposure to simple carbohydrates, causing rapid transit through the gastrointestinal tract. Manifestations include vertigo, tachycardia, syncope, sweating, pallor, and palpitations. This client does not have these reported manifestations.

Client was brought to the ED by the police after he started driving erratically and had almost collided with several vehicles. After being pulled over, the client stated he was 'driving into this future.' The client was incoherent in his responses to police officers and became angry when he was arrested. He is brought to the emergency department for medical clearance.

The client's impaired judgment is of significant concern because it threatens the client's and others' safety. The impairment in judgment can enable the client to do reckless actions (go through a stop sign, initiate a confrontation with others, indiscriminate sexual activity). The other findings are not significantly important because they do not present a safety issue compared to impaired judgment. A significant difference between hypomania and mania is its impairment in an individual's life. Hypomania causes an individual to be euphoric, have an expansive affect, and have the need for little sleep. A key difference is that hypomania typically lasts four days or less and does not impair an individual's life, whereas mania does cause an impairment. Additionally, mania may induce perceptual disturbances such as hallucinations. This is not a feature found in hypomania. Mania may require temporary hospitalization because of the lack of judgment and insight. The client is demonstrating mania as evidence by his erratic driving and impairment of cognition and judgement. This client will need to be admitted because of his risk of self-harm. This self-harm is linked to his inability to make appropriate judgments. This client's inability to make sound judgments makes him at risk for harm to himself and others. Thus, this client should be involuntarily admitted as the criteria for this type of admission is if the client is a danger to themselves or others. Valproic acid (VPA) needs to be administered to the client because it is a mood stabilizer. This medication has the goal to break the client's mania. Haloperidol would be inappropriate because this is indicated for psychotic disorders such as schizophrenia. Considering the client's volatile behavior, it is appropriate for the client to receive enhanced observation to minimize and risk of self-harm. Valproic acid (VPA) is a mood stabilizer and may cause blood dyscrasias such as thrombocytopenia; thus, monitoring the CBC is critical. VPA is also hepatotoxic; liver function tests should be observed for any liver injury. Arterial blood gas and urine electrolytes are unnecessary to monitor while a client takes VPA. VPA does not influence urine electrolytes and woul

Which five (5) client findings require follow-up by the nurse? Reports of epigastric pain Reports of shortness of breath Progressive worsening of symptoms Reports of dizziness Pale skin and diaphoresis

This client presented atypical signs of unstable acute coronary syndrome, as evidenced by the ST changes on the 12-lead electrocardiogram. The client reports epigastric pain, dizziness, and shortness of breath. These findings are an atypical presentation of ACS often found in women. Substernal chest pain with a gradual onset. Pain that radiates to the arm or jaw. Chest pain that is not relieved with rest. Diaphoresis and pallor may be additional findings. Atypical clinical features of ACS include - Nausea and vomiting Dyspnea Significant fatigue Epigastric pain Atypical features are found in women and individuals with diabetes mellitus. Individuals with diabetes mellitus have attenuated chest pain because of neuropathy. The client in this scenario is over the age of 50, which puts her at high risk of developing a myocardial infarction. The findings demonstrated by the client that require follow-up include the client reporting epigastric pain, shortness of breath, worsening of symptoms, dizziness, and diaphoresis, all consistent with atypical and unstable ACS. The client has a history of diabetes, and a blood glucose of 110 mg/dL would not be pertinent clinical findings related to ACS symptoms. However, the client being a woman and a diabetic make it likely for her to have atypical clinical features of ACS. It would be helpful for the nurse to gather a more comprehensive pain assessment, such as inquiring if the pain radiates anywhere. Pain radiating to the upper extremities is highly suggestive of ischemic pain. This is an essential question to ask for the nurse to narrow down if the client is experiencing unstable acute coronary syndrome. The other options would not validate acute coronary syndrome, which the client is experiencing.

The nurse is caring for a 14-year-old scheduled for an appendectomy. What is the nurse's role in obtaining informed consent before surgery? Select all that apply.

Validate that the parents are competent to provide consent for the client Witness the signature on the informed consent Choices C and D are correct. Since the client is 14, they are a minor, and their parents will be responsible for signing informed consent. The nurse is accountable for validating that the parents are competent to provide consent for the client (Choice C). The nurse will serve as the witness for the informed consent. This is one of the primary responsibilities of the nurse when a client is getting a procedure and signing a consent. The other primary responsibility will be to serve as the client's advocate and ensure that the parents have received sufficient information to make an informed decision. If they have not, the nurse must call the surgeon to return and speak further with the parents (Choice D). Choices A, B, and E are incorrect. The surgeon is not the only individual that may withdraw consent. If the client decides to withdraw their consent, they have that legal right. It is not within the nurses' scope of practice to review the risks and benefits of the surgery with the client. It is the surgeon's responsibility to perform this function. The nurse may clarify the general elements of surgery but cannot explain the risks, benefits, or alternatives. The consent does not need to be witnessed by two healthcare providers; one healthcare provider (such as a licensed practical nurse or registered nurse) can serve as the witness. The nurse only needs a second nurse if the nurse is witnessing telephone consent from a client's spouse or another immediate family member.

The nurse is caring for a client who was recently admitted to the cardiac floor for angina. This client states that their chest pain occurs at the same time every day at rest. The patient does not believe there are any precipitating factors. Which of the following types of angina is this patient most likely experiencing?

Variant angina Choice A is correct. Variant angina, also known as Prinzmetal's angina, occurs at about the same time every day, usually at rest. Variant angina is treated with calcium channel blockers. Choice B is incorrect. Stable angina occurs after activity and is relieved by nitroglycerin tablets. Choice C is incorrect. Unstable angina is less predictable and may precipitate myocardial infarction. Choice D is incorrect. This type of discomfort does not describe nonanginal pain. NCSBN client need Topic: Physiological Integrity, physiological adaptation

The nurse should administer the ceftriaxone in the client's ventrogluteal to prevent nerve damage The nurse should obtain a prescription for EMLA (lidocaine and prilocaine) cream to apply before the injection to decrease the child's pain.

When administering an intramuscular (IM) injection for an adult or a child, the dorsogluteal site should be avoided because the needle may pierce a nerve and cause damage. Thus, giving an IM ventrogluteal is acceptable. Prior to obtaining intravenous (IV) access or giving an IM injection, the nurse should obtain a prescription for EMLA (eutectic mixture of local anesthetics [lidocaine and prilocaine]) cream and apply it 60 minutes at the site to lessen the pain. ✓ To locate the ventrogluteal site, palpate to locate the greater trochanter, anterior superior iliac tubercle (found by flexing the thigh at the hip and measuring up to 1-2 cm [0.4-0.8 inch] above the crease formed in the groin), and posterior iliac crest; place palm over the greater trochanter, index finger over anterior superior iliac tubercle, and middle finger along the crest of ileum posteriorly as far as possible; inject into center of V formed by fingers. ✓ When treating bacterial otitis media by mouth (PO), antibiotics are commonly utilized. ✓ The advantage of IM ceftriaxone is a single injection which eliminates the possibility of non-adherence to the treatment plan.

A nurse is caring for a client receiving digoxin. The client's most recent digitalis level was 2.5 ng/mL. The nurse should take which action? Select all that apply.

Withhold the client's scheduled dose Assess the client's heart rate and rhythm Choices A and E are correct. The client's digitalis level of 2.5 ng/mL indicates toxicity. Digoxin has a narrow therapeutic index, which can cause significant side effects, such as cardiac arrhythmias (e.g., bradycardia, heart block, ventricular arrhythmias), even at plasma concentrations only twice the therapeutic plasma concentration range. Normal corrective serum digoxin levels range from 0.5-2 ng/mL. A level higher than 2 ng/mL is considered toxic. The nurse is correct in withholding the scheduled dose and assessing the client's heart rate and rhythm, as the client is likely to be experiencing bradycardia. Choices B, C, D, and F are incorrect. It would be incorrect to administer the next dose, as this would exacerbate the toxicity. Assessing the urinary output and sodium is not relative to digitalis toxicity and is not the priority here. A significant trigger in digitalis toxicity is hypokalemia, not hyponatremia. Notifying the physician regarding the toxic level is appropriate, but there is no reason to obtain an echocardiogram. An echocardiogram will not add any additional information at this point. Instead, an electrocardiogram must be obtained to look for any rhythm disturbances due to digoxin toxicity. The normal therapeutic range for digitalis is 0.5-2 ng/dL. Hypokalemia is a significant cause of digitalis toxicity and may be induced by certain diuretics. The earliest manifestation of digoxin toxicity is lack of appetite, nausea, and vomiting.

The nurse is assessing a group of clients prescribed lithium. The client at most significant risk for lithium toxicity is a client with

chronic migraine headaches and was newly prescribed naproxen. Choice B is correct. NSAIDs (naproxen, ibuprofen), ACE inhibitors (lisinopril, enalapril), and diuretics (furosemide, hydrochlorothiazide) should be avoided while a client is taking lithium. ACE inhibitors promote sodium wasting, and low levels of sodium precipitate lithium toxicity. NSAIDs reduce renal blood flow, cause lithium retention and raise its serum level to a potentially toxic range. The client with aches and pains should be recommended acetaminophen. Choices A, C, and D are incorrect. None of these conditions or medications pose a risk to a client taking lithium. Bronchodilators (salmeterol, albuterol) does not have a purported interaction with lithium. Clonidine is an appropriate antihypertensive for a client with lithium. Clonidine via a transdermal patch is kept on the client for seven days. Lithium may cause a client to develop hypothyroidism, and levothyroxine is safe.

The nurse is caring for a client diagnosed with peptic ulcer disease. The nurse anticipates a prescription for which medication? Incorrect

famotidine Choice C is correct. Famotidine is a histamine antagonist that prevents the parietal cells from secreting gastric acid. Indications for using famotidine include peptic ulcer disease, esophagitis, and gastric reflux. Other drugs in this class include cimetidine. Choices A, B, and D are incorrect. Ondansetron is an anti-emetic drug indicated in the management of nausea and vomiting. Diphenoxylate with atropine is an antidiarrhea medication that may be used in conditions such as irritable bowel syndrome. Psyllium is a bulk-forming laxative that is taken with plenty of water to ease constipation. This bulk-forming laxative may be added to the care plan for a client with diverticulosis. ADDITIONAL

The nurse cares for a client who sustained a stroke impacting the occipital lobe. Which of the following assessment findings would support this diagnosis? Incorrect

homonymous hemianopia Choice A is correct. Visual disturbances are expected for a client with a stroke impacting the occipital lobe of the brain. The occipital lobe is the primary optical center of the brain. Homonymous hemianopia is a complete left or right visual field defect. The client may need to be taught to scan the room, and the nurse should place objects in the unaffected visual field. Choices B, C, and D are incorrect. Proprioception is for the client to have an awareness of their body position. Testing a client's proprioception is done via the Romberg test. If the client should have impairments with proprioception, this highly suggests an insult to the parietal lobe, which primarily processes sensory input, proprioception, and taste. Expressive aphasia and impulsivity can be explained as an insult to the frontal lobe, which is the brain's executive center. Broca's area can be found in the frontal lobe and, if damaged, may cause the client to have expressive aphasia. Impulsivity is also a feature of a stroke that primarily affects the frontal lobe, as this lobe controls cognition, judgment, affective response to situations, and reasonable deduction. Frontal Lobe ✓ Regulates affective responses to situations ✓ Primary motor area; Broca's area for auditory expression ✓ Controls reasoning, concentration, cognition Temporal Lobe ✓ Wernecke area for language comprehension ✓ Interpretation of sound ✓ Complicated memory patterns Parietal Lobe ✓ Proprioception ✓ Processes nonverbal expressions ✓ Processes taste Occipital Lobe ✓ The primary visual center

The nurse is caring for assigned clients. The nurse should immediately follow-up on the client who

is being treated for pneumonia and develops restlessness. Choice A is correct. Restlessness is a concerning finding as it may indicate the client is experiencing shock or hypoxia. The nurse needs to act and perform vital signs to determine the etiology of restlessness. If the restlessness is due to hypoxia, supplemental oxygen should be provided. The most significant complication with pneumonia is the client developing acute respiratory distress syndrome, which may require mechanical ventilation. Choices B, C, and D are incorrect. Headache is a common finding for both dehydration and influenza. Considering this is an expected finding, the nurse should not prioritize this client over the client experiencing restlessness. Tidaling and intermittent bubbling in the water seal chamber of the chest drainage system are expected. It is continuous bubbling in this chamber, which indicates an air leak. Pleurisy is a condition that causes inflammation of the pleural cavity, and a classic sign is coughing, which induces chest pain and chest pain with inspiration. Treatment may be prescribed antibiotics and anti-inflammatories.

A client at 32 gestational weeks reports the sudden onset of painless, bright red vaginal bleeding. The assessment showed a normal fetal heart rate and a non-tender uterus. The nurse understands that this client is at the highest risk of developing

placenta previa. Choice A is correct. Placenta previa may occur as early as 20 gestational weeks. The manifestations of painless, bright red vaginal bleeding coincide with this condition. Commonly, the presentation of placenta previa is a finding on routine ultrasound examination at approximately 16 to 20 weeks. Choices B, C, and D are incorrect. A threatened abortion may only occur before 20 gestational weeks. Thus, this condition is excluded. Placental abruption is highly serious and manifests with painful vaginal bleeding that causes the uterus to be firm and tender. Uterine souffle is a soft, blowing sound. This sound may be auscultated over the uterus. This is the sound of blood circulating through the dilated uterine vessels, and it corresponds to the maternal pulse. Placenta previa is an implantation of the placenta in the lower uterus. As a result, the placenta is closer to the internal cervical os than to the presenting part (usually the head) of the fetus. The classic sign of placenta previa is the sudden onset of painless uterine bleeding in the last half of pregnancy. Many cases of placenta previa are diagnosed by ultrasound examination before any bleeding occurs.

A nurse is caring for a group of preoperative clients. Which client situation requires follow-up? A client

receiving dextrose 5% in water (D5W) and has a blood glucose of 266 mg/dL. reporting that they shaved their abdomen for their scheduled appendectomy. reporting difficulty with their last surgery, stating they got 'a really high fever'. reporting burning upon urination and increased urinary frequency. Choices B, C, D, and E are correct. These clients require follow-up. Preoperative (and postoperative) hyperglycemia is detrimental to optimal outcomes. This client has a glucose of 266 mg/dL, which is hyperglycemia. This client should also have the prescribed infusion of D5W questioned, as this solution would further increase the glucose. Clipping hair at the operative site is the best practice because it reduces the risk of surgical site infection. If shaving has to be done, it is completed immediately before the incision to reduce the chance of postoperative infection. The client stated that they got a high fever after their previous surgery and requires follow-up. This could be a concern for malignant hyperthermia. Although rare, this genetic disorder can be life-threatening when the client is exposed to certain anesthesia. The client reported burning upon urination preoperative and requires follow-up as preoperative infections may cause surgery cancelation as they complicate healing. Choice A is incorrect. This client does not require follow-up. Carbamazepine is an anticonvulsant and is commonly permitted to be taken with a sip of water to prevent seizure activity. When performing a preoperative surgical assessment, the nurse assesses the client's physical status and reviews elements such as Adherence to nothing by mouth (NPO) status Preoperative laboratory and diagnostic data Basic understanding of the procedure Discharge planning Postoperative education

The nurse is assessing a client experiencing serotonin syndrome. Which of the following findings would be expected? Select all that apply.

tachycardia altered mental status Choices A and B are correct. Tachycardia, altered mental status, increased blood pressure, myoclonus, hypertonia, and fever are manifestations of serotonin syndrome. Serotonin syndrome (or toxicity) is commonly caused by two serotonergic agents causing very high serotonin levels, which may lead to death in rare situations. The causes of serotonin syndrome primarily stem from the client being exposed to excessive serotonergic agents, including tramadol, SSRIs (citalopram, paroxetine, etc.), and MAOIs. Choices C, D, and E are incorrect. The client experiencing serotonin syndrome is hyperactive, which explains the tachycardia and high blood pressure. Thus, somnolence (drowsiness) would not be expected as the client would likely be experiencing psychomotor agitation. Anuria would not be expected; this is when the client is not voiding. This is not associated with serotonin syndrome. The client experiencing experiencing serotonin syndrome would be experiencing hyperactivity and this activation would raise blood pressure and heart rate. ✓ Serotonin syndrome (toxicity) is caused when the client overdoses on a serotonergic agent or has a drug-to-drug adverse interaction (such as taking an MAOI and an SSRI) ✓ Example serotonergic drugs include citalopram, tramadol, venlafaxine, paroxetine, etc. ✓ No lab test is available to diagnosis this syndrome and a medication history and thorough physical examination is used to support diagnosis ✓ Manifestations of this syndrome include psychomotor agitation, increased blood pressure, tachycardia, restlessness, hypertonia, abdominal pain, altered mental status, and fever ✓ Treatment for serotonin syndrome includes prescribed benzodiazepines to temper the psychomotor agitation. Additionally, supportive care with prescribed cooling measures, parenteral fluids, and supplemental oxygen may be utilized.


Related study sets

Chapter 47: Assessment- Endocrine System

View Set

Chapter 13 - Reformations & Religious Wars

View Set

Unit 3 (chapter 9) History of Graphic Design

View Set